Mental Health Practice HESI

Lakukan tugas rumah & ujian kamu dengan baik sekarang menggunakan Quizwiz!

71. A client with bipolar disorder on the mental health unit becomes loud, and shouts at one of the nurses, "You fat tub of lard! Get something done around here!" What is the best initial action for the nurse to take? A) Have the orderly escort the client to his room. B) Tell the client his healthcare provider will be notified if he continues to be verbally abusive. C) Redirect the client's energy by asking him to tidy the recreation room. D) Call the healthcare provider to obtain a prescription for a sedative.

Distracting the client, or redirecting his energy (C), prevents further escalation of the inappropriate behavior. (A) could result in escalating the abuse and unnecessarily involve another staff member in the abusive situation. (B) is a threat and is using a health team member (healthcare provider) as the threat. (D) may be indicated if the behavior escalates, but, at this time, the best initial action is (C). Correct Answer(s): C

44. A 35-year-old male client who has been hospitalized for two weeks for chronic paranoia continues to state that someone is trying to steal his clothing. Which action should the nurse implement? A) Encourage the client to actively participate in assigned activities on the unit. B) Place a lock on the client's closet. C) Ignore the client's paranoid ideation to extinguish these behaviors. D) Explain to the client that his suspicions are false.

Diverting the client's attention from paranoid ideation and encouraging him to complete assignments can be helpful in assisting him to develop a positive self-image (A). The client's problem is not security, and (B) actually supports his paranoid ideation. (C) is not correct because ignoring the client's symptoms may lower his self-esteem. The nurse should not argue with the client about his delusions (D), and should not try to reason with the client regarding his paranoid ideation. Correct Answer(s): A

5. A client who is being treated with lithium carbonate for bipolar disorder develops diarrhea, vomiting, and drowsiness. What action should the nurse take? A) Notify the healthcare provider immediately and prepare for administration of an antidote. B) Notify the healthcare provider of the symptoms prior to the next administration of the drug. C) Record the symptoms as normal side effects and continue administration of the prescribed dosage. D) Hold the medication and refuse to administer additional amounts of the drug.

Early side effects of lithium carbonate (occurring with serum lithium levels below 2.0 mEq per liter) generally follow a progressive pattern beginning with diarrhea, vomiting, drowsiness, and muscular weakness. At higher levels, ataxia, tinnitus, blurred vision, and large dilute urine output may occur. (B) is the best choice. Although these are expected symptoms, the healthcare provider should be notified prior to the next administration of the drug. (A, C, and D) would not reflect good nursing judgment. Correct Answer(s): B

35. A male client is admitted to the psychiatric unit with a medical diagnosis of paranoid schizophrenia. During the admission procedure, the client looks up and states, "No, it's not MY fault. You can't blame me. I didn't kill him, you did." What action is best for the nurse to take? A) Reassure the client by telling him that his fear of the admission procedure is to be expected. B) Tell the client that no one is accusing him of murder and remind him that the hospital is a safe place. C) Assess the content of the hallucinations by asking the client what he is hearing. D) Ignore the behavior and make no response at all to his delusional statements.

Further assessment is indicated (C). The nurse should obtain information about what the client believes the voices are telling him--they may be telling him to kill the nurse! (A) is telling the client how he feels (fearful). The nurse should leave communications open and seek more information. (B) is arguing with the client's delusion, and the nurse should never argue with a client's hallucinations or delusions, also (B) is possibly offering false reassurance. (D) is avoiding the situation and the client's needs. Correct Answer(s): C

27. A homeless person who is in the manic phase of bipolar disorder is admitted to the mental health unit. Which laboratory finding obtained on admission is most important for the nurse to report to the healthcare provider? A) Decreased thyroid stimulating hormone level. B) Elevated liver function profile. C) Increased white blood cell count. D) Decreased hematocrit and hemoglobin levels.

Hyperthyroidism causes an increased level of serum thyroid hormones (T3 and T4), which inhibit the release of TSH (A), so the client's manic behavior may be related to an endocrine disorder. (B, C, and D) are abnormal findings that are commonly found in the homeless population because of poor sanitation, poor nutrition, and the prevalence of substance abuse. Correct Answer(s): A

70. A client on the psychiatric unit appears to imitate a certain nurse on the unit. The client seeks out this particular nurse and imitates her mannerisms. The nurse knows that the client is using which defense mechanism? A) Sublimation. B) Identification. C) Introjection. D) Repression.

Identification (B) is an attempt to be like someone or emulate the personality traits of another. (A) is substituting an unacceptable feeling for one that is more socially acceptable. (C) is incorporating the values or qualities of an admired person or group into one's own ego structure. (D) is the involuntary exclusion of painful thoughts or memories from one's awareness. Correct Answer(s): B

64. Within several days of hospitalization, a client is repeatedly washing the top of the same table. Which initial intervention is best for the nurse to implement to help the client cope with anxiety related to this behavior? A) Administer a prescribed PRN antianxiety medication. B) Assist the client to identify stimuli that precipitates the ritualistic activity. C) Allow time for the ritualistic behavior, then redirect the client to other activities. D) Teach the client relaxation and thought stopping techniques.

Initially, the nurse should allow time for the ritual (C) to prevent anxiety. (A) may help reduce the client's anxiety, but will not prevent ritualistic behavior resulting from the client's ineffective coping ability. (B) is a long-term goal of individual therapy, but is not directly related to controlling the behavior at this time. (D) lists techniques that can be used to assist the client in learning new ways of interrupting obsessive thoughts and resulting ritualistic behavior as treatment progresses. Correct Answer(s): C

29. A male client is admitted to a mental health unit on Friday afternoon and is very upset on Sunday because he has not had the opportunity to talk with the healthcare provider. Which response is best for the nurse to provide this client? A) Let me call and leave a message for your healthcare provider. B) The healthcare provider should be here on Monday morning. C) How can I help answer your questions? D) What concerns do you have at this time?

It is best for the nurse to call the healthcare provider (A) because clients have the right to information about their treatment. Suggesting that the healthcare provider will be available the following day (B) does not provide immediate reassurance to the client. The nurse can also implement offer to assist the client (C and D), but the highest priority intervention is contacting the healthcare provider. Correct Answer(s): A

17. An 86-year-old female client with Alzheimer's disease is wandering the busy halls of the extended care facility and asks the nurse, "Where should I stand for the parade?" Which response is best for the nurse to provide? A) Anywhere you want to stand as long as you do not get hurt by those in the parade. B) You are confused because of all the activity in the hall. There is no parade. C) Let us go back to the activity room and see what is going on in there. D) Remember I told you that this is a nursing home and I am your nurse.

It is common for those with Alzheimer's disease to use the wrong words. Redirecting the client (using an accepting non-judgmental dialogue) to a safer place and familiar activities (C) is most helpful because clients experience short-term memory loss. (A) dismisses the client's attempt to find order and does not help her relate to her surroundings. (B) dismisses the client and may increase her anxiety level because it merely labels the client's behavior and offers no solution. It is very frustrating for those with Alzheimer's disease to "remember," and scolding them (D) may hurt their feelings. Correct Answer(s): C

10. The nurse observes a female client with schizophrenia watching the news on TV. She begins to laugh softly and says, "Yes, my love, I'll do it." When the nurse questions the client about her comment she states, "The news commentator is my lover and he speaks to me each evening. Only I can understand what he says." What is the best response for the nurse to make? A) What do you believe the news commentator said to you? B) Let's watch news on a different television channel. C) Does the news commentator have plans to harm you or others? D) The news commentator is not talking to you.

It is imperative that the nurse determine what the client believes she heard (A). The idea of reference may be to hurt herself or someone else, and the main function of a psychiatric nurse is to maintain safety. (B) is acceptable, but it is best to determine the client's beliefs. (C) is validating the idea of reference, while (D) is challenging the client. Correct Answer(s): A

67. A nurse working in the emergency room of a children's hospital admits a child whose injuries could have resulted from abuse. Which statement most accurately describes the nurse's responsibility in cases of suspected child abuse? A) The nurse should obtain objective data such as x-rays before reporting suspicions to the authorities. B) The nurse should confirm any suspicions of child abuse with the healthcare provider before reporting to the authorities. C) The nurse should report any case of suspected child abuse to the nurse in charge. D) The nurse should note in the client's record any suspicions of child abuse so that a history of such suspicions can be tracked.

It is the nurse's legal responsibility to report all suspected cases of child abuse. Notifying the charge nurse starts the legal reporting process (C). Correct Answer(s): C

6. The parents of a 14-year-old boy bring their son to the hospital. He is lethargic, but responsive. The mother states, "I think he took some of my pain pills." During initial assessment of the teenager, what information is most important for the nurse to obtain from the parents? A) If he has seemed depressed recently. B) If a drug overdose has ever occurred before. C) If he might have taken any other drugs. D) If he has a desire to quit taking drugs.

Knowledge of all substances taken (C) will guide further treatment, such as administration of antagonists, so obtaining this information has the highest priority. (A and B) are also valuable in planning treatment. (D) is not appropriate during the acute management of a drug overdose. Correct Answer(s): C

50. A client who is known to abuse drugs is admitted to the psychiatric unit. Which medication should the nurse anticipate administering to a client who is exhibiting benzodiazepine withdrawal symptoms? A) Perphenazine (Trilafon). B) Diphenhydramine (Benadryl). C) Chlordiazepoxide (Librium). D) Isocarboxazid (Marplan).

Librium (C), an antianxiety drug, as well as other benzodiazepines, are used in titrated doses to reduce the severity of abrupt benzodiazepine withdrawal. (A) is an antipsychotic agent. (B) is an antihistamine and antianxiety drug. (D) is an MAO inhibitor. Correct Answer(s): C

56. When preparing a teaching plan for a client who is to be discharged with a prescription for lithium carbonate (Lithonate), it is most important for the nurse to include which instruction? A) It may take 3 to 4 weeks to achieve therapeutic effects. B) Keep your dietary salt intake consistent. C) Avoid eating aged cheese and chicken liver. D) Eat foods high in fiber such as whole grain breads.

Lithium's effectiveness is influenced by salt intake (B). Too much salt causes more lithium to be excreted, thereby decreasing the effectiveness of the drug. Too little salt causes less lithium to be excreted, potentially resulting in toxicity. (A, C, and D) are not specific instructions pertinent to teaching about lithium carbonate (Lithonate). Correct Answer(s): B

33. Which statement about contemporary mental health nursing practice is accurate? A) There is one approved theoretical framework for psychiatric nursing practice. B) Psychiatric nursing has yet to be recognized as a core mental health discipline. C) Contemporary practice of psychiatric nursing is primarily focused on inpatient care. D) The psychiatric nursing client may be an individual, family, group, organization, or community.

Mental health nursing is not only concerned with one-on-one interactions. Psychiatric stressors can impact and be reflected in the overall direction, activities, and responses involving families, groups, and entire communities (D). (A, B, and C) are incorrect statements about the status of mental health nursing. Correct Answer(s): D

38. The nurse is assessing the parents of a nuclear family who are attending a support group for parents of adolescents. According to Erikson, these parents who are adapting to middle adulthood should exhibit which characteristic? A) Loss of independence. B) Increased self-understanding. C) Isolation from society. D) Development of intimate relationships.

Middle adulthood is characterized by self-reflection, understanding, and acceptance (B), and generativity or guidance of children. (A and C) are maladaptive behaviors in middle adulthood. Although middle-aged adults may delay or re-establish intimate relationships, (D) is initially developed during young adulthood. Correct Answer(s): B

53. A client is admitted with a diagnosis of depression. The nurse knows that which characteristic is most indicative of depression? A) Grandiose ideation. B) Self-destructive thoughts. C) Suspiciousness of others. D) A negative view of self and the future.

Negative self-image and feelings of hopelessness about the future (D) are specific indicators for depression. (A and/or C) occurs with paranoia or paranoid ideation. (B) may be seen in depressed clients, but are not always present, so (D) is a better answer than (B). Correct Answer(s): D

72. A client who has been admitted to the psychiatric unit tells the nurse, "My problems are so bad that no one can help me." Which response is best for the nurse to make? A) How can I help? B) Things probably aren't as bad as they seem right now. C) Let's talk about what is right with your life. D) I hear how miserable you are, but things will get better soon.

Offering self shows empathy and caring (A), and is the best of the choices provided. Combining the first part of (D) with (A) would be the best response, but this is not a fill-in-the-blank or an essay test! Choose the best of those choices provided and move on. (B) dismisses the client, things are bad as far as this client is concerned. (C) avoids the client's problems and promotes denial. "I hear how miserable you are" is an example of reflective dialogue and would be the best choice if it were not for the rest of the sentence--"but things will get better" which is offering false reassurance. Correct Answer(s): A

13. Which diet selection by a client who is depressed and taking the MAO inhibitor tranylcypromine sulfate (Parnate) indicates to the nurse that the client understands the dietary restrictions imposed by this medication regimen? A) Hamburger, French fries, and chocolate milkshake. B) Liver and onions, broccoli, and decaffeinated coffee. C) Pepperoni and cheese pizza, tossed salad, and a soft drink. D) Roast beef, baked potato with butter, and iced tea.

Only (D) contains no tyramine. Tyramine in foods interacts with MAOI in the body causing a hypertensive crisis which is life-threatening, and Parnate is classified as an MAOI antidepressant. Some items in (A, B, and C) contain tyramine and would not be permitted for a client taking Parnate. Correct Answer(s): D

42. On admission to a residential care facility, an elderly female client tells the nurse that she enjoys cooking, quilting, and watching television. Twenty-fours after admission, the nurse notes that the client is withdrawn and isolated. It is best for the nurse to encourage this client to become involved in which activity? A) Clean the unit kitchen cabinets. B) Participate in a group quilting project. C) Watch television in the activity room. D) Bake a cake for a resident's birthday.

Peer interaction in a group activity (B) will help to prevent social isolation and withdrawal. (A, C, and D) are activities that can be accomplished alone, without peer interaction. Correct Answer(s): B

1. A male client with schizophrenia who is taking fluphenazine decanoate (Prolixin decanoate) is being discharged in the morning. A repeat dose of medication is scheduled for 20 days after discharge. The client tells the nurse that he is going on vacation in the Bahamas and will return in 18 days. Which statement by the client indicates a need for health teaching? A) When I return from my tropical island vacation, I will go to the clinic to get my Prolixin injection. B) While I am on vacation and when I return, I will not eat or drink anything that contains alcohol. C) I will notify the healthcare provider if I have a sore throat or flu-like symptoms. D) I will continue to take my benztropine mesylate (Cogentin) every day.

Photosensitivity is a side effect of Prolixin and a vacation in the Bahamas (with its tropical island climate) increases the client's chance of experiencing this side effect. He should be instructed to avoid direct sun (A) and wear sunscreen. (B, C, and D) indicate accurate knowledge. Alcohol acts synergistically with Prolixin (B). (C) lists signs of agranulocytosis, which is also a side effect of Prolixin. In order to avoid extrapyramidal symptoms (EPS), anticholinergic drugs, such as Cogentin, are often prescribed prophylactically with Prolixin. Correct Answer(s): A

46. A child is brought to the emergency room with a broken arm. Because of other injuries, the nurse suspects the child may be a victim of abuse. When the nurse tries to give the child an injection, the child's mother becomes very loud and shouts, "I won't leave my son! Don't you touch him! You'll hurt my child!" What is the best interpretation of the mother's statements? The mother is A) regressing to an earlier behavior pattern. B) sublimating her anger. C) projecting her feelings onto the nurse. D) suppressing her fear.

Projection is attributing one's own thoughts, impulses, or behaviors onto another--it is the mother who is probably harming the child and she is attributing her actions to the nurse (C). The mother may be immature, but (A) is not the best description of her behavior. (B) is substituting a socially acceptable feeling for an unacceptable one. These are not socially acceptable feelings. The mother may be suppressing her fear (D) by displaying anger, but such an interpretation cannot be concluded from the data presented. Correct Answer(s): C

51. A 35-year-old male client on the psychiatric ward of a general hospital believes that someone is trying to poison him. The nurse understands that a client's delusions are most likely related to his A) early childhood experiences involving authority issues. B) anger about being hospitalized. C) low self-esteem. D) phobic fear of food.

Psychotic clients have difficulty with trust and have low self-esteem (C). Nursing care should be directed at building trust and promoting positive self-esteem. Activities with limited concentration and no competition should be encouraged in order to build self-esteem. (A, B, and D) are not specifically related to the development of delusions. Correct Answer(s): C

21. A 30-year-old sales manager tells the nurse, "I am thinking about a job change. I don't feel like I am living up to my potential." Which of Maslow's developmental stages is the sales manager attempting to achieve? A) Self-Actualization. B) Loving and Belonging. C) Basic Needs. D) Safety and Security.

Self-actualization is the highest level of Maslow's development stages, which is an attempt to fulfill one's full potential (C). (B) is identifying support systems. (C) is the first level of Maslow's developmental stages and is the foundation upon which higher needs rest. Individuals who feel safe and secure (D) in their environment perceive themselves as having physical safety and lack fear of harm. Correct Answer(s): A

54. A woman brings her 48-year-old husband to the outpatient psychiatric unit and describes his behavior to the admitting nurse. She states that he has been sleepwalking, cannot remember who he is, and exhibits multiple personalities. The nurse knows that these behaviors are often associated with A) dissociative disorder. B) obsessive-compulsive disorder. C) panic disorder. D) post-traumatic stress syndrome.

Sleepwalking, amnesia, and multiple personalities are examples of detaching emotional conflict from one's consciousness, which is the definition of a dissociative disorder (A). (B) is characterized by persistent, recurrent intrusive thoughts or urges (obsessions) that are unwilled and cannot be ignored, and provoke impulsive acts (compulsions) such as handwashing. (C) is an acute attack of anxiety characterized by personality disorganization. (D) is re-experiencing a psychologically terrifying or distressing event that is outside the usual range of human experience, such as war, rape, etc. Correct Answer(s): A

15. The nurse is assessing a client's intelligence. Which factor should the nurse remember during this part of the mental status exam? A) Acute psychiatric illnesses impair intelligence. B) Intelligence is influenced by social and cultural beliefs. C) Poor concentration skills suggests limited intelligence. D) The inability to think abstractly indicates limited intelligence.

Social and cultural beliefs (B) have significant impact on intelligence. Chronic psychiatric illness may impair intelligence (A), especially if it remains untreated. Limited concentration does not suggest limited intelligence (C). Difficulties with abstractions are suggestive of psychotic thinking (D), not limited intelligence. Correct Answer(s): B

65. A 72-year-old female client is admitted to the psychiatric unit with a diagnosis of major depression. Which statement by the client should be of greatest concern to the nurse and require further assessment? A) I will die if my cat dies. B) I don't feel like eating this morning. C) I just went to my friend's funeral. D) Don't you have more important things to do?

Sometimes a client will use an analogy to describe themselves, and (A) would be an indication for conducting a suicide assessment. (B) could have a variety of etiologies, and while further assessment is indicated, this statement does not indicate potential suicide. Normal grief process differs from depression, and at this client's age peer/cohort deaths are more frequent, so (C) would be within normal limits. (D) is an expression of low self-esteem typical of depression. (B, C, and D) are examples of decreased energy and mood levels which would negate suicide ideation at this time. Correct Answer(s): A

49. The nurse is planning care for a 32-year-old male client diagnosed with HIV infection who has a history of chronic depression. Recently, the client's viral load has begun to increase rather than decrease despite his adherence to the HIV drug regimen. What should the nurse do first while taking the client's history upon admission to the hospital? A) Determine if the client attends a support group weekly. B) Hold all antidepressant medications until further notice. C) Ask the client if he takes St. John's Wort routinely. D) Have the client describe any recent changes in mood.

St. John's Wort, an herbal preparation, is an alternative (nonconventional) therapy for depression, but it may adversely interact with medications used to treat HIV infection (C). The nurse's top priority upon admission is to determine if the client has been taking this herb concurrently with HIV antiviral drugs, which may explain the rise in the viral load. Asking about (A or D) may be helpful in gathering more data about the client's depressive state, but these issues do not have the priority of (C). (B) may be harmful to the client. Correct Answer(s): C

69. A client is receiving substitution therapy during withdrawal from benzodiazepines. Which expected outcome statement has the highest priority when planning nursing care? A) Client will not demonstrate cross-addiction. B) Co-dependent behaviors will be decreased. C) Excessive CNS stimulation will be reduced. D) Client's level of consciousness will increase.

Substitution therapy with another CNS depressant is intended to decrease the excessive CNS stimulation that can occur during benzodiazepine withdrawal (C). (A, B, and D) are all appropriate outcome statements for the client described, but do not have the priority of (C). Correct Answer(s): C

12. The nurse is planning discharge for a male client with schizophrenia. The client insists that he is returning to his apartment, although the healthcare provider informed him that he will be moving to a boarding home. What is the most important nursing diagnosis for discharge planning? A) Ineffective denial related to situational anxiety. B) Ineffective coping related to inadequate support. C) Social isolation related to difficult interactions. D) Self-care deficit related to cognitive impairment.

The best nursing diagnosis is (A) because the client is unable to acknowledge the move to a boarding home. (B, C, and D) are potential nursing diagnoses, but denial is most important because it is a defense mechanism that keeps the client from dealing with his feelings about living arrangements. Correct Answer(s): A

31. A female client with depression attends group and states that she sometimes misses her medication appointments because she feels very anxious about riding the bus. Which statement is the nurse's best response? A) Can your case manager take you to your appointments? B) Take your medication for anxiety before you ride the bus. C) Let's talk about what happens when you feel very anxious. D) What are some ways that you can cope with your anxiety?

The best response is to explore ways for the client to cope with anxiety (D). The nurse should encourage problem-solving rather than dependence on the case manager (A) for transportation. Strategies for coping with anxiety should be encouraged before suggesting (B). (C) is therapeutic, but the best response is an open-ended question to explore ways to cope with the anxiety. Correct Answer(s): D

22. A male client with mental illness and substance dependency tells the mental health nurse that he has started using illegal drugs again and wants to seek treatment. Since he has a dual diagnosis, which person is best for the nurse to refer this client to first? A) The emergency room nurse. B) His case manager. C) The clinic healthcare provider. D) His support group sponsor.

The case manager (B) is responsible for coordinating community services, and since this client has a dual diagnosis, this is the best person to describe available treatment options. (A) is unnecessary, unless the client experiences behaviors that threaten his safety or the safety of others. (C and D) might also be useful, but it is most important at this time that a treatment program be coordinated to meet this client's needs. Correct Answer(s): B

26. The nurse observes a client who is admitted to the mental health unit and identifies that the client is talking continuously, using words that rhyme but that have no context or relationship with one topic to the next in the conversation. This client's behavior and thought processes are consistent with which syndrome? A) Dementia. B) Depression. C) Schizophrenia. D) Chronic brain syndrome.

The client is demonstrating symptoms of schizophrenia (C), such as disorganized speech that may include word salad (communication that includes both real and imaginary words in no logical order), incoherent speech, and clanging (rhyming). Dementia (A) is a global impairment of intellectual (cognitive) functions that may be progressive, such as Alzheimer's or organic brain syndrome (D). Depression (C) is typified by psychomotor retardation, and the client appears to be slowed down in movement, in speech, and would appear listless and disheveled. Correct Answer(s): C

34. A 40-year-old male client diagnosed with schizophrenia and alcohol dependence has not had any visitors or phone calls since admission. He reports he has no family that cares about him and was living on the streets prior to this admission. According to Erikson's theory of psychosocial development, which stage is the client in at this time? A) Isolation. B) Stagnation. C) Despair. D) Role confusion.

The client is in Erikson's "Generativity vs. Stagnation" stage (age 24 to 45), and meeting the task includes maintaining intimate relationships and moving toward developing a family (B). (A) occurs in young adulthood (age 18 to 25), (C) occurs in maturity (age 45 to death), and (D) occurs in adolescence (age 12 to 20). These are all stages that occur if individuals are not successfully coping with their psychosocial developmental stage. Correct Answer(s): B

25. A client, who is on a 30-day commitment to a drug rehabilitation unit, asks the nurse if he can go for a walk on the grounds of the treatment center. When he is told that his privileges do not include walking on the grounds, the client becomes verbally abusive. Which approach should the nurse use? A) Call a staff member to escort the client to his room. B) Tell the client to talk to his healthcare provider about his privileges. C) Remind the client of the unit rules. D) Ignore the client's inappropriate behavior.

The client is trying to engage the nurse in a dispute. Ignoring the behavior (D) provides no reinforcement for the inappropriate behavior. (A) is not necessary unless the client becomes a physical threat to the nurse. (B) would be inappropriate, because it is referring the situation to the healthcare provider and is not in keeping with good health team management. Consistent limits must be established and enforced. (C) would subject the nurse to more verbal abuse because the client could use any response as an excuse to attack the nurse once again. Correct Answer(s): D

73. A 52-year-old male client in the intensive care unit who has been oriented suddenly becomes disoriented and fearful. Assessment of vital signs and other physical parameters reveal no significant change and the nurse formulates the diagnosis, "Confusion related to ICU psychosis." Which intervention is best to implement? A) Move all machines away from the client's immediate area. B) Attempt to allay the client's fears by explaining the etiology of his condition. C) Cluster care so that brief periods of rest can be scheduled during the day. D) Extend visitation times for family and friends.

The critical care environment confronts clients with an environment which provides stressors heightened by treatment modalities that may prove to be lifesaving. These stressors can result in isolation and confusion. The best intervention is to provide the client with rest periods (C). (A) is not practical--the machinery is often lifesaving. The client is not ready for (B). Although family and friends (D) can provide a support system to the client, visits should be limited because of the critical care that must be provided. Correct Answer(s): C

66. The nurse is conducting discharge teaching for a client with schizophrenia who plans to live in a group home. Which statement is most indicative of the need for careful follow-up after discharge? A) Crickets are a good source of protein. B) I have not heard any voices for a week. C) Only my belief in God can help me. D) Sometimes I have a hard time sitting still.

The most frequent cause of increased symptoms in psychotic clients is non-compliance with the medication regimen. If clients believe that "God alone" is going to heal them (C), then they may discontinue their medication, so (C) would pose the greatest threat to this client's prognosis. (A) would require further teaching, but is not as significant a statement as (C). (B) indicates an improvement in the client's condition. (D) may be a sign of anxiety that could improve with treatment, but does not have the priority of (C). Correct Answer(s): C

41. A female client refuses to take an oral hypoglycemic agent because she believes that the drug is being administered as part of an elaborate plan by the Mafia to harm her. Which nursing intervention is most important to include in this client's plan of care? A) Reassure the client that no one will harm her while she is in the hospital. B) Ask the healthcare provider to give the client the medication. C) Explain that the diabetic medication is important to take. D) Reassess client's mental status for thought processes and content.

The most important intervention is to reassess the client's mental status (D) and to take further action based on the findings of this assessment. Attempting to reassure the client (A) is in effect arguing with the client's delusions and could escalate an already anxious situation. Collaborating about diabetic care (B and C) is not likely to help change the client's false beliefs. Correct Answer(s): D

2. A male client is admitted to the mental health unit because he was feeling depressed about the loss of his wife and job. The client has a history of alcohol dependency and admits that he was drinking alcohol 12 hours ago. Vital signs are: temperature, 100° F, pulse 100, and BP 142/100. The nurse plans to give the client lorazepam (Ativan) based on which priority nursing diagnosis? A) Risk for injury related to suicidal ideation. B) Risk for injury related to alcohol detoxification. C) Knowledge deficit related to ineffective coping. D) Health seeking behaviors related to personal crisis.

The most important nursing diagnosis is related to alcohol detoxification (B) because the client has elevated vital signs, a sign of alcohol detoxification. Maintaining client safety related to (A) should be addressed after giving the client Ativan for elevated vital signs secondary to alcohol withdrawal. (C and D) can be addressed when immediate needs for safety are met. Correct Answer(s): B

8. The community health nurse talks to a male client who has bipolar disorder. The client explains that he sleeps 4 to 5 hours a night and is working with his partner to start two new businesses and build an empire. The client stopped taking his medications several days ago. What nursing problem has the highest priority? A) Excessive work activity. B) Decreased need for sleep. C) Medication management. D) Inflated self-esteem.

The most important nursing problem is medication management (C) because compliance with the medication regimen will help prevent hospitalization. The client is also exhibiting signs of (A, B, and C); however, these problems do not have the priority of medication management. Correct Answer(s): C

3. The charge nurse is collaborating with the nursing staff about the plan of care for a client who is very depressed. What is the most important intervention to implement during the first 48 hours after the client's admission to the unit? A) Monitor appetite and observe intake at meals. B) Maintain safety in the client's milieu. C) Provide ongoing, supportive contact. D) Encourage participation in activities.

The most important reason for closely observing a depressed client immediately after admission is to maintain safety (B), since suicide is a risk with depression. (A, C, and D) are all important interventions, but safety is the priority. Correct Answer(s): B

7. The wife of a male client recently diagnosed with schizophrenia asks the nurse, "What exactly is schizophrenia? Is my husband all right?" Which response is best for the nurse to provide to this family member? A) It sounds like you're worried about your husband. Let's sit down and talk. B) It is a chemical imbalance in the brain that causes disorganized thinking. C) Your husband will be just fine if he takes his medications regularly. D) I think you should talk to your husband's psychologist about this question.

The nurse should answer the client's question with factual information and explain that schizophrenia is a chemical imbalance in the brain (B). (A) is a therapeutic response but does not answer the question, and may be an appropriate response after the nurse answers the question asked. Although (C) is likely true to some degree, it is also true that some clients continue to have disorganized thinking even with antipsychotic medications. Referring the spouse to the psychologist (D) is avoiding the issue; the nurse can and should answer the question. Correct Answer(s): B

45. A nurse working on a mental health unit receives a community call from a person who is tearful and states, "I just feel so nervous all of the time. I don't know what to do about my problems. I haven't been able to sleep at night and have hardly eaten for the past 3 or 4 days." The nurse should initiate a referral based on which assessment? A) Altered thought processes. B) Moderate levels of anxiety. C) Inadequate social support. D) Altered health maintenance.

The nurse should initiate a referral based on anxiety levels (B) and feelings of nervousness that interfere with sleep, appetite, and the inability to solve problems. The client does not report symptoms of (A) or evidence of (C). There is not enough information to initiate a referral based on (D). Correct Answer(s): B

20. A male client with schizophrenia tells the nurse that the voices he hears are saying, "You must kill yourself." To assist the client in coping with these thoughts, which response is best for the nurse to provide? A) Tell yourself that the voices are unreasonable. B) Exercise when you hear the voices. C) Talk to someone when you hear the voices. D) The voices aren't real, so ignore them.

The nurse should teach the client to use self-talk to disprove the voices (A). Although (B) may be helpful, the client's concrete thinking may make it difficult to understand this suggestion. Clients with schizophrenia have difficulty initiating interaction with others (C). Auditory hallucinations are often relentless, so it is difficult to ignore them (D). Correct Answer(s): A

9. At a support meeting of parents of a teenager with polysubstance dependency, a parent states, "Each time my son tries to quit taking drugs, he gets so depressed that I'm afraid he will commit suicide." The nurse's response should be based on which information? A) Addiction is a chronic, incurable disease. B) Tolerance to the effects of drugs causes feelings of depression. C) Feelings of depression frequently lead to drug abuse and addiction. D) Careful monitoring should be provided during withdrawal from the drugs.

The priority is to teach the parents that their son will need monitoring and support during withdrawal (D) to ensure that he does not attempt suicide. Although (A and C) are true, they are not as relevant to the parent's expressed concern. There is no information to support (B). Correct Answer(s): D

47. An anxious client expressing a fear of people and open places is admitted to the psychiatric unit. What is the most effective way for the nurse to assist this client? A) Plan an outing within the first week of admission. B) Distract her whenever she expresses her discomfort about being with others. C) Confront her fears and discuss the possible causes of these fears. D) Accompany her outside for an increasing amount of time each day.

The process of gradual desensitization by controlled exposure to the situation which is feared (D), is the treatment of choice in phobic reactions. (A and C) are far too aggressive for the initial treatment period and could even be considered hostile. (B) promotes denial of the problem, and gives the client the message that discussion of the phobia is not permitted. Correct Answer(s): D

61. A 46-year-old female client has been on antipsychotic neuroleptics for the past three days. She has had a decrease in psychotic behavior and appears to be responding well to the medication. On the fourth day, the client's blood pressure increases, she becomes pale and febrile, and demonstrates muscular rigidity. Which action should the nurse initiate? A) Place the client on seizure precautions and monitor carefully. B) Immediately transfer the client to ICU. C) Describe the symptoms to the charge nurse and record on the client's chart. D) No action is required at this time as these are known side effects of such drugs.

These symptoms are descriptive of neuroleptic malignant syndrome (NMS) which is an extremely serious/life threatening reaction to neuroleptic drugs (B). The major symptoms of this syndrome are fever, rigidity, autonomic instability, and encephalopathy. Respiratory failure, cardiovascular collapse, arrhythmias, and/or renal failure can result in death. This is an EMERGENCY situation, and the client requires immediate critical care. Seizure precautions (A) are not indicated in this situation. (C and D) do not consider the seriousness of the situation. Correct Answer(s): B

43. A male adolescent is admitted with bipolar disorder after being released from jail for assault with a deadly weapon. When the nurse asks the teen to identify his reason for the assault, he replies, "Because he made me mad!" Which goal is best for the nurse to include in the client's plan of care? The client will A) outline methods for managing anger. B) control impulsive actions toward self and others. C) verbalize feelings when anger occurs. D) recognize consequences for behaviors exhibited.

Those with bipolar disorder often exhibit poor impulse control, and the most important goal for this client at this time is to learn to control impulsive behavior (B) so that he can avert the social consequences related to such behaviors. (A, C, and D) are important goals, but they do not address the acute issue of impulse control, which is necessary to reduce the likelihood of harming self or others. Correct Answer(s): B

14. An elderly female client with advanced dementia is admitted to the hospital with a fractured hip. The client repeatedly tells the staff, "Take me home. I want my Mommy." Which response is best for the nurse to provide? A) Orient the client to the time, place, and person. B) Tell the client that the nurse is there and will help her. C) Remind the client that her mother is no longer living. D) Explain the seriousness of her injury and need for hospitalization.

Those with dementia often refer to home or parents when seeking security and comfort. The nurse should use the techniques of "offering self" and "talking to the feelings" to provide reassurance (B). Clients with advanced dementia have permanent physiological changes in the brain (plaques and tangles) that prevent them from comprehending and retaining new information, so (A, C, and D) are likely to be of little use to this client and do not help the client's emotional needs. Correct Answer(s): B

4. A 38-year-old female client is admitted with a diagnosis of paranoid schizophrenia. When her tray is brought to her, she refuses to eat and tells the nurse, "I know you are trying to poison me with that food." Which response is most appropriate for the nurse to make? A) I'll leave your tray here. I am available if you need anything else. B) You're not being poisoned. Why do you think someone is trying to poison you? C) No one on this unit has ever died from poisoning. You're safe here. D) I will talk to your healthcare provider about the possibility of changing your diet.

(A) is the best choice cited. The nurse does not argue with the client nor demand that she eat, but offers support by agreeing to "be there if needed", e.g., to warm the food. (B and C) are arguing with the client's delusions, and (B) asks "why" which is usually not a good question for a psychotic client. (D) has nothing to do with the actual problem; i.e., the problem is not the diet (she thinks any food given to her is poisoned.) Correct Answer(s): A

16. The nurse should include which interventions in the plan of care for a severely depressed client with neurovegetative symptoms? (Select all that apply.) A) Permit rest periods as needed. B) Speaking slowly and simply. C) Place the client on suicide precautions. D) Allow the client extra time to complete tasks. E) Observe and encourage food and fluid intake. F) Encourage mild exercise and short walks on the unit

(A, B, D, E, and F) should be included in this client's plan of care because these measures promote the client's comfort and well-being. Neurovegetative symptoms accompany the mood disorder of depression and include physiological disruptions, such as anorexia, constipation, sleep disturbance, and psychomotor retardation. Suicidal ideation (C) does not usually accompany the neurovegetative state because the client does not have the energy or high level of anxiety associated with a suicide attempt. Correct Answer(s): A, B, D, E, F

68. Physical examination of a 6-year-old reveals several bite marks in various locations on his body. X-ray examination reveals healed fractures of the ribs. The mother tells the nurse that her child is always having accidents. Which initial response by the nurse would be most appropriate? A) I need to inform the healthcare provider about your child's tendency to be accident prone. B) Tell me more specifically about your child's accidents. C) I must report these injuries to the authorities because they do not seem accidental. D) Boys this age always seem to require more supervision and can be quite accident prone.

(B) seeks more information using an open ended, non-threatening statement. (A) could be appropriate, but it is not the best answer because the nurse is being somewhat sarcastic and is also avoiding the situation by referring it to the healthcare provider for resolution. Although it is true that suspected cases of child abuse must be reported, (C) is virtually an attack and is jumping to conclusions before conclusive data has been obtained. (D) is a cliché and dismisses the seriousness of the situation. Correct Answer(s): B

37. A 45-year-old male client tells the nurse that he used to believe that he was Jesus Christ, but now he knows he is not. Which response is best for the nurse to make? A) Did you really believe you were Jesus Christ? B) I think you're getting well. C) Others have had similar thoughts when under stress. D) Why did you think you were Jesus Christ?

(C) offers support by assuring the client that others have suffered as he has (also the principle on which Alcoholics Anonymous acts). (A) is belittling. (B) is making an inappropriate judgment. You may have narrowed your choices to (C and D). However, you should eliminate (D) because it is a "why" question, and the client does not know why! Correct Answer(s): C

39. A 25-year-old female client has been particularly restless and the nurse finds her trying to leave the psychiatric unit. She tells the nurse, "Please let me go! I must leave because the secret police are after me." Which response is best for the nurse to make? A) No one is after you, you're safe here. B) You'll feel better after you have rested. C) I know you must feel lonely and frightened. D) Come with me to your room and I will sit with you.

(D) is the best response because it offers support without judgment or demands. (A) is arguing with the client's delusion. (B) is offering false reassurance. (C) is a violation of therapeutic communication in that the nurse is telling the client how she feels (frightened and lonely), rather than allowing the client to describe her own feelings. Hallucinating and/or delusional clients are not capable of discussing their feelings, particularly when they perceive a crisis. Correct Answer(s): D

32. The nurse is leading a "current events group" with chronic psychiatric clients. One group member states, "Saddam Hussein was my nurse during my last hospitalization. He was a very mean nurse and wasn't nice to me." Which response is best for the nurse to make? A) Saddam Hussein was not your nurse. B) What did he do to you that was so mean? C) I didn't know that Saddam Hussein was a nurse. D) I agree that Saddam Hussein is not a very nice man.

(D) presents the reality of the situation (the individual is not nice) in relation to American culture. The fact that Saddam Hussein is not a nurse should be addressed on an individual basis. Since this is group therapy, the nurse would be illustrating the concept of universality. (A) is likely to promote defensiveness. (B and C) would support the delusion. Correct Answer(s): D

52. The nurse plans to help an 18-year-old female mentally retarded client ambulate the first postoperative day after an appendectomy. When the nurse tells the client it is time to get out of bed, the client becomes angry and tells the nurse, "Get out of here! I'll get up when I'm ready!" Which response is best for the nurse to make? A) Your healthcare provider has prescribed ambulation on the first postoperative day. B) You must ambulate to avoid complications which could cause more discomfort than ambulating. C) I know how you feel. You're angry about having to ambulate, but this will help you get well. D) I'll be back in 30 minutes to help you get out of bed and walk around the room.

(D) provides a "cooling off" period, is firm, direct, non-threatening, and avoids arguing with the client. (A) is avoiding responsibility by referring to the healthcare provider. (B) is trying to reason with a mentally retarded client and is threatening the client with "complications." (C) is telling the client how she feels (angry), and the nurse does not really "know" how this client feels, unless the nurse is mentally retarded and has just had an appendectomy! Correct Answer(s): D

48. The nurse suspects child abuse when assessing a 3-year-old boy and noticing several small, round burns on his legs and trunk that might be the result of cigarette burns. Which parental behavior provides the greatest validation for such suspicions? A) The parents' explanation of how the burns occurred is different from the child's explanation of how they occurred. B) The parents seem to dismiss the severity of the child's burns, saying they are very small and have not posed any problem. C) The parents become very anxious when the nurse suggests that the child may need to be admitted for further evaluation. D) The parents tell the nurse that the child was burned in a house fire which is incompatible with the nurse's observation of the type of burn.

(D) provides the most validation. The parent's explanation (subjective data) is incompatible with the objective data (small round burns on the legs and trunk). (A) provides only subjective data, and the child's explanation could be influenced by factors such as age, fear, or imagination. The parent's apparent lack of concern (B) is inconclusive, but the nurse's opinion of the parents' reaction is subjective and could be wrong. (C) might provide a clue that child abuse occurred, but the nurse must remember that most parents are anxious about their child being hospitalized. Correct Answer(s): D

Which are characteristics of the termination stage of group development? Select all that apply. 1. The group evaluates the experience. 2. The real work of the group is accomplished. 3. Group interaction involves superficial conversation. 4. Group members become acquainted with each other. 5. Some structuring of group norms, roles, and responsibilities takes place. 6. The group explores members' feelings about the group and the impending separation.

1 and 6 Rationale: The stages of group development include the initial stage, the working stage, and the termination stage. During the initial stage, the group members become acquainted with each other, and some structuring of group norms, roles, and responsibilities takes place. During the initial stage, group interaction involves superficial conversation. During the working stage, the real work of the group is accomplished. During the termination stage, the group evaluates the experience and explores members' feelings about the group and the impending separation.

The nurse is preparing a client for electroconvulsive therapy (ECT). The family of the client asks the nurse about this treatment. The nurse responds, knowing that which statements are accurate regarding this treatment? Select all that apply. 1. The average series involves 6 to 12 treatments. 2. Some confusion may be noted after the procedure. 3. Memory loss will occur but will resolve with time. 4. This treatment is a permanent cure to the condition. 5. This treatment is tried before the use of medications.

1, 2, 3 Rationale: ECT as a form of treatment is considered when medication therapy has failed, the client is at high risk for suicide, or depression is judged to be overwhelmingly severe. Treatments are administered three times a week, with an average series involving 8 to 12 treatments over a duration of 2 to 4 weeks. The most common side effect is amnesia for events occurring near the period of treatment. Memory deficits may occur and tend to resolve with time. This treatment is not a permanent cure to the client's condition.

The nurse is preparing a client for electroconvulsive therapy (ECT), which is scheduled for the next morning. Which interventions would be included in the preprocedural plan? Select all that apply. 1. Obtain an informed consent. 2. Have the client void before the procedure. 3. Remove dentures and contact lenses before the procedure. 4. Withhold food and fluids for 6 hours before the treatment. 5. Administer tap water enemas on the evening before the procedure.

1, 2, 3, 4 Rationale: Enemas are not a component of the pretreatment care for a client scheduled for ECT. Options 1, 2, 3, and 4 are a part of the pretreatment plan. Additionally, the nurse should teach the client and family what to expect with ECT and allow the client to discuss his or her feelings regarding the procedure.

The nurse is developing a plan of care for a client with depression whose food intake is poor. The nurse should include which interventions in the plan of care? Select all that apply. 1. Assist the client in selecting foods from the food menu. 2. Offer high-calorie fluids throughout the day and evening. 3. Allow the client to eat alone in the room if the client requests to do so. 4. Offer small high-calorie, high-protein snacks during the day and evening. 5. Select the foods for the client to be sure that the client eats a balanced diet.

1, 2, 4 Rationale: In caring for a client with depression whose nutritional intake is poor, the nurse should remain with the client during the meal. The nurse also should assist the client in selecting foods from the menu because the client is more likely to eat the foods that he or she likes. Offering small high-calorie, high-protein snacks and high-calorie fluids throughout the day and evening are appropriate interventions for the client to maintain nutrition.

The nurse in the mental health unit recognizes which as being therapeutic communication techniques? Select all that apply. 1. Restating 2. Listening 3. Asking the client, "Why?" 4. Maintaining neutral responses 5. Providing acknowledgment and feedback 6. Giving advice and approval or disapproval

1, 2, 4, 5 Rationale: Therapeutic communication techniques include listening, maintaining silence, maintaining neutral responses, using broad openings and open-ended questions, focusing and refocusing, restating, clarifying and validating, sharing perceptions, reflecting, providing acknowledgment and feedback, giving information, presenting reality, encouraging formulation of a plan of action, providing nonverbal encouragement, and summarizing. Asking why is often interpreted as being accusatory by the client and should also be avoided. Providing advice or giving approval or disapproval are barriers to communication.

The nurse is preparing to perform an admission assessment on a client with a diagnosis of bulimia nervosa. Which assessment findings does the nurse expect to note? Select all that apply. 1. Dental decay 2. Moist oily skin 3. Loss of tooth enamel 4. Electrolyte imbalances 5. Body weight well below ideal range

1, 3, 4 Rationale: Clients with bulimia nervosa initially may not appear to be physically or emotionally ill. They are often at or slightly below ideal body weight. On further inspection, a client exhibits dental decay and loss of tooth enamel if the client has been inducing vomiting. Electrolyte imbalances are present. Dry, scaly skin (rather than moist, oily skin) is present.

Which nursing interventions are appropriate for a hospitalized client with mania who is exhibiting manipulative behavior? Select all that apply. 1. Communicate expected behaviors to the client. 2. Ensure that the client knows that they are not in charge of the nursing unit. 3. Assist the client in identifying ways of setting limits on personal behaviors. 4. Follow through about the consequences of behavior in a nonpunitive manner. 5. Enforce rules by informing the client that they will not be allowed to attend therapy groups. 6. Have the client state the consequences for behaving in ways that are viewed as unacceptable.

1, 3, 4, 6 Rationale: Interventions for dealing with the client exhibiting manipulative behavior include setting clear, consistent, and enforceable limits on manipulative behaviors; being clear with the client regarding the consequences of exceeding limits set; following through with the consequences in a nonpunitive manner; and assisting the client in identifying means of setting limits on personal behaviors. Ensuring that the client knows that he or she is not in charge of the nursing unit is inappropriate; power struggles need to be avoided. Enforcing rules and informing the client that he or she will not be allowed to attend therapy groups is a violation of a client's rights.

Which interventions are most appropriate for caring for a client in alcohol withdrawal? Select all that apply. 1. Monitor vital signs. 2. Maintain NPO status. 3. Provide a safe environment. 4. Address hallucinations therapeutically. 5. Provide stimulation in the environment. 6. Provide reality orientation as appropriate.

1, 3, 4, 6 Rationale: When the client is experiencing withdrawal from alcohol, the priority for care is to prevent the client from harming self or others. The nurse would provide a low-stimulation environment to maintain the client in as calm a state as possible. The nurse would monitor the vital signs closely and report abnormal findings. The nurse would reorient the client to reality frequently and would address hallucinations therapeutically. Adequate nutritional and fluid intake need to be maintained.

Which statement made by an unlicensed assistive personnel (UAP) indicates to the registered nurse that the UAP understands the concepts related to suicide? 1. "Discussing suicide with a client is not harmful." 2. "Those clients who talk about suicide never do it." 3. "Depressed clients are the only persons who commit suicide." 4. "When a person talks about making suicide threats, the only thing the person wants is attention from family and friends."

1. "Discussing suicide with a client is not harmful." Rationale: An open discussion of suicide will not encourage a client to make a decision to commit suicide and in fact often will help to prevent it. Such a discussion offers the health care professional the opportunity to assess the reality of suicide for the client and take necessary precautions to keep the client safe. Options 2, 3, and 4 are inaccurate statements regarding suicide.

The nurse determines that the wife of an alcoholic client is benefiting from attending an Al-Anon group if the nurse hears the wife make which statement? 1. "I no longer feel that I deserve the beatings my husband inflicts on me." 2. "My attendance at the meetings has helped me to see that I provoke my husband's violence." 3. "I enjoy attending the meetings because they get me out of the house and away from my husband." 4. "I can tolerate my husband's destructive behaviors now that I know they are common with alcoholics."

1. "I no longer feel that I deserve the beatings my husband inflicts on me." Rationale: Al-Anon support groups are a protected, supportive opportunity for spouses and significant others to learn what to expect and to obtain excellent pointers about successful behavioral changes. The correct option is the healthiest response because it exemplifies an understanding that the alcoholic partner is responsible for his behavior and cannot be allowed to blame family members for loss of control. Option 2 is incorrect because the nonalcoholic partner should not feel responsible when the spouse loses control. Option 3 indicates that the group is viewed as an escape, not as a place to work on issues. Option 4 indicates that the wife remains codependent.

A client is planning to attend Overeaters Anonymous. Which statement by the client indicates a need for additional information regarding this self-help group? 1. "The leader is a nurse or psychiatrist." 2. "The members provide support to each other." 3. "People who have a similar problem are able to help others." 4. "It is designed to serve people who have a common problem."

1. "The leader is a nurse or psychiatrist." Rationale: The sponsor of a self-help group is an experienced member of the group. The nurse or psychiatrist may be asked by the group to serve as a resource, but would not be the leader of the group. The remaining options are characteristics of a self-help group.

A client with depression is scheduled to receive three sessions of electroconvulsive therapy (ECT). The client asks the nurse about the length of time it will take for improvement in the condition. The nurse should tell the client he or she will see improvement approximately how long after the three treatments? 1. 1 week 2. 3 weeks 3. 4 weeks 4. 8 weeks

1. 1 week Rationale: Health care providers generally administer ECT treatments three times a week, with an average series including 8 to 12 treatments. After three sessions of ECT, the client should start to demonstrate improvement in 1 week. Options 2, 3, and 4 are incorrect.

Which client is most at risk for committing suicide? 1. A 75-year-old client with metastatic cancer 2. A 71-year-old client with a cardiac disorder 3. A 24-year-old client who just had an argument with her roommate 4. A 30-year-old newly divorced client who states she has custody of the children

1. A 75-year-old client with metastatic cancer Rationale: The person most at risk for suicide is the client with terminal illness. Other high-risk groups include adolescents, drug abusers, persons who have experienced recent losses, those who have few or no social supports, and those with a history of suicide attempts and a suicide plan.

A client is preparing to attend a Gamblers Anonymous meeting for the first time. The nurse should tell the client that which is the first step in this 12-step program? 1. Admitting to having a problem 2. Substituting other activities for gambling 3. Stating that the gambling will be stopped 4. Discontinuing relationships with people who gamble

1. Admitting to having a problem Rationale: The first step in the 12-step program is to admit that a problem exists. Substituting other activities for gambling may be a strategy but it is not the first step. The remaining options are not realistic strategies for the initial step in a 12-step program.

A client with diabetes mellitus is told that amputation of the leg is necessary to sustain life. The client is very upset and tells the nurse, "This is all my health care provider's fault. I have done everything I've been asked to do!" Which nursing interpretation is best for this situation? 1. An expected coping mechanism 2. An ineffective coping mechanism 3. A need to notify the hospital lawyer 4. An expression of guilt on the part of the client

1. An expected coping mechanism Rationale: The nurse needs to be aware of the effective and ineffective coping mechanisms that can occur in a client when loss is anticipated. The expression of anger is known to be a normal response to impending loss, and the anger may be directed toward the self, God or other spiritual being, or caregivers. Notifying the hospital lawyer is inappropriate. Guilt may or may not be a component of the client's feelings, and the data in the question do not indicate that guilt is present.

The home health nurse visits a client at home and determines that the client is dependent on drugs. During the assessment, which action should the nurse take to plan appropriate nursing care? 1. Ask the client why he started taking illegal drugs. 2. Ask the client about the amount of drug use and its effect. 3. Ask the client how long he thought that he could take drugs without someone finding out. 4. Not ask any questions for fear that the client is in denial and will throw the nurse out of the home.

1. Ask the client why he started taking illegal drugs. Rationale: Whenever the nurse carries out an assessment for a client who is dependent on drugs, it is best for the nurse to attempt to elicit information by being nonjudgmental and direct. Option 1 is incorrect because it is judgmental and off-focus, and reflects the nurse's bias. Option 3 is incorrect because it is judgmental, insensitive, and aggressive, which is nontherapeutic. Option 4 is incorrect because it indicates passivity on the nurse's part and uses rationalization to avoid the therapeutic nursing intervention.

A hospitalized client with a history of alcohol abuse tells the nurse, "I am leaving now. I have to go. I don't want any more treatment. I have things that I have to do right away." The client has not been discharged and is scheduled for an important diagnostic test to be performed in 1 hour. After the nurse discusses the client's concerns with the client, the client dresses and begins to walk out of the hospital room. What action should the nurse take? 1. Call the nursing supervisor. 2. Call security to block all exit areas. 3. Restrain the client until the health care provider (HCP) can be reached. 4. Tell the client that the client cannot return to this hospital again if the client leaves now.

1. Call the nursing supervisor. Rationale: Most health care facilities have documents that the client is asked to sign relating to the client's responsibilities when the client leaves against medical advice. The client should be asked to wait to speak to the HCP before leaving and to sign the "against medical advice" document before leaving. If the client refuses to do so, the nurse cannot hold the client against the client's will. Therefore, in this situation, the nurse should call the nursing supervisor. The nurse can be charged with false imprisonment if a client is made to believe wrongfully that he or she cannot leave the hospital. Restraining the client and calling security to block exits constitutes false imprisonment. All clients have a right to health care and cannot be told otherwise.

A home care nurse making an initial home visit notes that a client is taking donepezil hydrochloride (Aricept). The nurse questions the client's spouse about a history of which disorder that is treated with this medication? 1. Dementia 2. Schizophrenia 3. Seizure disorder 4. Obsessive-compulsive disorder

1. Dementia Rationale: Donepezil hydrochloride is a cholinergic agent that is used in the treatment of mild to moderate dementia of the Alzheimer's type. It increases concentration of acetylcholine, which slows the progression of Alzheimer's disease. The other options are incorrect and are not indications for use of this medication.

A client being seen in the emergency department immediately after being sexually assaulted appears calm and controlled. The nurse analyzes this behavior as indicating which defense mechanism? 1. Denial 2. Projection 3. Rationalization 4. Intellectualization

1. Denial Rational: Denial is refusal to admit to a painful reality and may be a response by a victim of sexual abuse. In this case the client is not acknowledging the trauma of the assault either verbally or nonverbally. Projection is transferring one's internal feelings, thoughts, and unacceptable ideas and traits to someone else. Rationalization is justifying the unacceptable attributes about oneself. Intellectualization is the excessive use of abstract thinking or generalizations to decrease painful thinking.

A client admitted to a mental health unit for treatment of psychotic behavior spends hours at the locked exit door shouting, "Let me out. There's nothing wrong with me. I don't belong here." What defense mechanism is the client implementing? 1. Denial 2. Projection 3. Regression 4. Rationalization

1. Denial Rationale: Denial is refusal to admit to a painful reality, which is treated as if it does not exist. In projection, a person unconsciously rejects emotionally unacceptable features and attributes them to other persons, objects, or situations. Regression allows the client to return to an earlier, more comforting, although less mature, way of behaving. Rationalization is justifying illogical or unreasonable ideas, actions, or feelings by developing acceptable explanations that satisfy the teller and the listener.

The emergency department nurse is caring for an adult client who is a victim of family violence. Which priority instruction should be included in the discharge instructions? 1. Information regarding shelters 2. Instructions regarding calling the police 3. Instructions regarding self-defense classes 4. Explaining the importance of leaving the violent situation

1. Information regarding shelters Rationale: Tertiary prevention of family violence includes assisting the victim after the abuse has already occurred. The nurse should provide the client with information regarding where to obtain help, including a specific plan for removing the self from the abuser and information regarding escape, hotlines, and the location of shelters. An abused person is usually reluctant to call the police. Teaching the victim to fight back is not the appropriate action for the victim when dealing with a violent person. Explaining the importance of leaving the violent situation is important, but a specific plan is necessary.

The nurse has been observing a client closely who has been displaying aggressive behaviors. The nurse observes that the behavior displayed by the client is escalating. Which nursing intervention is least helpful to this client at this time? 1. Initiate confinement measures. 2. Acknowledge the client's behavior. 3. Assist the client to an area that is quiet. 4. Maintain a safe distance from the client.

1. Initiate confinement measures. Rationale: During the escalation period, the client's behavior is moving toward loss of control. Nursing actions include taking control, maintaining a safe distance, acknowledging behavior, moving the client to a quiet area, and medicating the client if appropriate. To initiate confinement measures during this period is inappropriate. Initiation of confinement measures, if needed, is most appropriate during the crisis period.

The nurse understands that which best describes Gestalt therapy? 1. It emphasizes self-expression, self-exploration, and self-awareness in the present. 2. It promotes the individual's comfort in the group, which then transfers to other relationships. 3. The therapist focuses on how irrational beliefs and thoughts contribute to psychological distress. 4. The therapist's goal is to help others express their feelings toward one another during group sessions.

1. It emphasizes self-expression, self-exploration, and self-awareness in the present. Rationale: Gestalt therapy emphasizes self-expression, self-exploration, and self-awareness in the present. The client and therapist focus on everyday problems and try to solve them. Interpersonal group therapy promotes the individual's comfort in the group, which then transfers to other relationships. In rational emotive therapy, the therapist focuses on how irrational beliefs and thoughts contribute to psychological distress. In Rogerian therapy, the therapist's goal is to help others express their feelings toward one another during group sessions.

Which type of therapeutic approach has the characteristic that all team members are seen as equally important in helping clients meet their goals? 1. Milieu therapy 2. Interpersonal therapy 3. Behavior modification 4. Rational emotive therapy

1. Milieu therapy Rationale: All treatment team members are viewed as significant and valuable to the client's successful treatment outcomes in milieu therapy. Interpersonal therapy is based on a one-to-one or group therapy approach in which the therapist-client relationship is often used as a way for the client to examine other relationships in his or her life. Behavior modification is based on rewards and punishment. Rational emotive therapy deals with the correction of distorted thinking.

When reviewing the admission assessment, the nurse notes that a client was admitted to the mental health unit involuntarily. Based on this type of admission, the nurse should provide which intervention for this client? 1. Monitor closely for harm to self or others. 2. Assist in completing an application for admission. 3. Supply the client with written information about their mental illness. 4. Provide an opportunity for the family to discuss why they felt the admission was needed.

1. Monitor closely for harm to self or others. Rationale: Involuntary admission is necessary when a person is a danger to self or others or is in need of psychiatric treatment regardless of the client's willingness to consent to the hospitalization. A written request is a component of a voluntary admission. Providing written information regarding the illness is likely premature initially. The family may have had no role to play in the client's admission.

The home care nurse is visiting an older client whose spouse died 6 months ago. Which behavior by the client indicates ineffective coping? 1. Neglecting personal grooming 2. Looking at old snapshots of family 3. Participating in a senior citizens' program 4. Visiting their spouse's grave once a month

1. Neglecting personal grooming Rational: Coping mechanisms are behaviors used to decrease stress and anxiety. In response to a death, ineffective coping is manifested by an extreme behavior that in some cases may be harmful to the individual physically or psychologically. The correct option is indicative of a behavior that identifies an ineffective coping behavior in the grieving process.

The nurse is planning care for a client being admitted to the nursing unit who attempted suicide. Which priority nursing intervention should the nurse include in the plan of care? 1. One-to-one suicide precautions 2. Suicide precautions with 30-minute checks 3. Checking the whereabouts of the client every 15 minutes 4. Asking the client to report suicidal thoughts immediately

1. One-to-one suicide precautions Rationale: One-to-one suicide precautions are required for a client who has attempted suicide. Options 2 and 3 may be appropriate, but not at the present time, considering the situation. Option 4 also may be an appropriate nursing intervention, but the priority is identified in the correct option. The best intervention is constant supervision so that the nurse may intervene as needed if the client attempts to harm himself or herself.

The nurse is caring for a client who is at risk for suicide. What is the priority nursing action for this client? 1. Provide authority, action, and participation. 2. Display an attitude of detachment, confrontation, and efficiency. 3. Demonstrate confidence in the client's ability to deal with stressors. 4. Provide hope and reassurance that the problems will resolve themselves.

1. Provide authority, action, and participation. Rationale: A crisis is an acute, time-limited state of disequilibrium resulting from situational, developmental, or societal sources of stress. A person in this state is temporarily unable to cope with or adapt to the stressor by using previous coping mechanisms. The person who intervenes in this situation (the nurse) "takes over" for the client (authority) who is not in control and devises a plan (action) to secure and maintain the client's safety. When this has occurred, the nurse works collaboratively with the client (participates) in developing new coping and problem-solving strategies.

The nurse observes that a client is pacing, agitated, and presenting aggressive gestures. The client's speech pattern is rapid, and affect is belligerent. Based on these observations, what is the nurse's immediate priority of care? 1. Provide safety for the client and other clients on the unit. 2. Provide the clients on the unit with a sense of comfort and safety. 3. Assist the staff in caring for the client in a controlled environment. 4. Offer the client a less stimulating area to calm down in and gain control.

1. Provide safety for the client and other clients on the unit. Rationale: Safety of the client and other clients is the priority. The correct option is the only one that addresses the safety needs of the client as well as those of the other clients.

The nurse is conducting a group therapy session. During the session, a client diagnosed with mania consistently disrupts the group's interactions. Which intervention should the nurse initially implement? 1. Setting limits on the client's behavior 2. Asking the client to leave the group session 3. Asking another nurse to escort the client out of the group session 4. Telling the client that they will not be able to attend any future group sessions

1. Setting limits on the client's behavior Rationale: Manic clients may be talkative and can dominate group meetings or therapy sessions by their excessive talking. If this occurs, the nurse initially would set limits on the client's behavior. Initially, asking the client to leave the session or asking another person to escort the client out of the session is inappropriate. This may agitate the client and escalate the client's behavior further. Barring the client from group sessions is also an inappropriate action because it violates the client's right to receive treatment and is a threatening action.

Which behavior observed by the nurse indicates a suspicion that a depressed adolescent client may be suicidal? 1. The adolescent gives away a DVD and a cherished autographed picture of a performer. 2. The adolescent runs out of the therapy group, swearing at the group leader, and runs to her room. 3. The adolescent becomes angry while speaking on the telephone and slams down the receiver. 4. The adolescent gets angry with her roommate when the roommate borrows the client's clothes without asking.

1. The adolescent gives away a DVD and a cherished autographed picture of a performer. Rationale: A depressed suicidal client often gives away that which is of value as a way of saying goodbye and wanting to be remembered. Options 2, 3, and 4 deal with anger and acting-out behaviors that are often typical of any adolescent.

A mental health nurse notes that a client with schizophrenia is exhibiting an immobile facial expression and a blank look. Which should the nurse document in the client's record? 1. The client has a flat affect. 2. The client has an inappropriate affect. 3. The client is exhibiting bizarre behavior. 4. The client's emotional responses exhibit a blunted affect.

1. The client has a flat affect. Rationale: A flat affect is manifested as an immobile facial expression or blank look. An inappropriate affect refers to an emotional response to a situation that is incongruent with the tone of the situation. A bizarre affect such as grimacing, laughing, and self-directed mumbling is marked when the client is unable to relate logically to the environment. A blunted affect is a minimal emotional response or outward affect that typically does not coincide with the client's inner emotions.

The nurse is performing an assessment on a client with dementia. Which data gathered during the assessment indicates a manifestation associated with dementia? 1. Uses confabulation 2. Improvement in sleeping 3. Absence of sundown syndrome 4. Presence of personal hygienic care

1. Uses confabulation Rationale: The clinical picture of dementia ranges from mild cognitive deficits to severe, life-threatening alterations in neurological functioning. For the client to use confabulation or the fabrication of events or experiences to fill in memory gaps is not unusual. Often, lack of inhibitions on the part of the client may constitute the first indication of something being "wrong" to the client's significant others (e.g., the client may undress in front of others, or the formerly well-mannered client may exhibit slovenly table manners). As the dementia progresses, the client will have difficulty sleeping and episodes of wandering or sundowning.

A client experiencing disturbed thought processes believes that his food is being poisoned. Which communication technique should the nurse use to encourage the client to eat? 1. Using open-ended questions and silence 2. Sharing personal preference regarding food choices 3. Documenting reasons why the client does not want to eat 4. Offering opinions about the necessity of adequate nutrition

1. Using open-ended questions and silence Rationale: Open-ended questions and silence are strategies used to encourage clients to discuss their problems. Sharing personal food preferences is not a client-centered intervention. The remaining options are not helpful to the client because they do not encourage the client to express feelings. The nurse should not offer opinions and should encourage the client to identify the reasons for the behavior.

The nurse is developing a plan of care for the client with a diagnosis of paranoia and should include which interventions in the plan of care? Select all that apply. 1. Provide a warm approach to the client. 2. Ask permission before touching the client. 3. Eliminate physical contact with the client. 4. Defuse any anger or verbal attacks with a nondefensive stance. 5. Use simple and clear language when communicating with the client.

2, 3, 4, 5 Rationale: When caring for a client with paranoia, the nurse should ask permission if touch is necessary because touch may be interpreted as a sexual or physical assault. The nurse must eliminate any physical contact and not touch the client. The anger that a paranoid client expresses often is displaced, and when a staff member becomes defensive, both client and staff anger may escalate. Simple and clear language should be used in speaking to the client to prevent misinterpretation and to clarify the nurse's intent and action. The nurse should avoid a warm approach because warmth can be frightening to a person who needs emotional distance.

A client is admitted with a recent history of severe anxiety following a home invasion and robbery. During the initial assessment interview, which statement by the client would indicate to the nurse the possible diagnosis of posttraumatic stress disorder? Select all that apply. 1. "I'm afraid of spiders." 2. "I keep reliving the robbery." 3. "I see his face everywhere I go." 4. "I don't want anything to eat now." 5. "I might have died over a few dollars in my pocket." 6. "I have to wash my hands over and over again many times."

2, 3, 5 Rationale: Reliving an event, experiencing emotional numbness (facing possible death), and having flashbacks of the event (seeing the same face everywhere) are all common occurrences with posttraumatic stress disorder. The statement. "I'm afraid of spiders," is more relative to having a phobia. The statement "I have to wash my hands over and over again many times" describes ritual compulsive behaviors to decrease anxiety for someone with obsessive compulsive disorder. Stating "I don't want anything to eat now" is vague and could relate to numerous conditions.

The nurse is performing an admission assessment on a client at high risk for suicide. The nurse should prepare to ask the client which assessment question to elicit data related to this risk? 1. "What are you feeling right now?" 2. "Do you have a plan to commit suicide?" 3. "How many times have you attempted suicide in the past?" 4. "Why were your attempts at suicide unsuccessful in the past?"

2. "Do you have a plan to commit suicide?" Rationale: When assessing for suicide risk, the nurse must determine if the client has a suicide plan. Clients who have a definitive plan pose a greater risk for suicide. Although options 1, 3, and 4 are questions that may provide information that will be helpful in planning care for the client, these questions will not provide information regarding the risk of suicide.

The nurse in the mental health unit is having a conversation with a client diagnosed with posttraumatic stress disorder. The client seems upset and looks anxious. What is the appropriate nursing statement the nurse should make to the client? 1. "Don't worry so much." 2. "I can see that you are upset." 3. "Everything is going to be all right." 4. "Why are you having so much trouble controlling your anxiety?"

2. "I can see that you are upset." Rationale: The correct option is the only one that addresses the client's feelings and concerns. Options 1 and 3 provide false reassurance and place the client's feelings on hold. Option 4 is a nontherapeutic communication technique and will increase the client's anxiety.

A nursing student is assisting with the care of a client with a chronic mental illness. The nurse informs the student that a behavior modification approach (operant conditioning) will be used in treatment for the client. Which statement by the student indicates a need for further information about the therapy? 1. "It uses positive reinforcement." 2. "It uses negative reinforcement." 3. "It increases social behaviors in the client." 4. "It increases the level of self-care in the client."

2. "It uses negative reinforcement." Rationale: Operant conditioning entails rewarding a client for desired behaviors and is the basis for behavior modification. It uses a positive reinforcement approach. Options 1, 3, and 4 are accurate characteristics of this form of therapy.

A mental health nurse in a psychiatric unit is meeting with a client who has a long history of acting out and violent behavior. The client also is known to have abused drugs on numerous occasions. During the session the client says to the nurse, "I'm feeling much better now, and I'm ready to go straight." Which response by the nurse would be therapeutic? 1. "You have said this many times before!" 2. "Tell me what makes you feel that you are ready." 3. "I have not seen any changes in you to believe that you are ready to go straight." 4. "I'm so glad to hear you talking this way. I will let your health care provider know."

2. "Tell me what makes you feel that you are ready." Rationale: Clients with a long history of acting out and violent behavior and those who have used drugs need to demonstrate motivation to change the behavior, not just verbalization of the behavior. The therapeutic response by the nurse would be directed at assisting the client to look at the behaviors that indicate the change. The correct option is the only one that will provide this direction to the client.

The spouse of a client admitted to the mental health unit for alcohol withdrawal says to the nurse, "I should get out of this bad situation." What is the most helpful response by the nurse? 1. "Why don't you tell your wife about this?" 2. "What do you find difficult about this situation?" 3. "This is not the best time to make that decision." 4. "I agree with you. You should get out of this situation."

2. "What do you find difficult about this situation?" Rationale: The most helpful response is one that encourages the client to solve problems. Giving advice implies that the nurse knows what is best and can foster dependency. The nurse should not agree with the client, and the nurse should not request that the client provide explanations.

The nurse in the mental health unit is assigned to care for a female client with a diagnosis of acute depression. In communicating with the client, which statement would be appropriate for the nurse to make? 1. "You look lovely today." 2. "You're wearing a new blouse." 3. "Don't worry-everyone gets depressed once in a while." 4. "You will feel better when your medication starts to work."

2. "You're wearing a new blouse." Rationale: A client who is depressed sees the negative side of everything. Telling the client that she looks lovely today can be interpreted as "didn't look lovely last time we met." Neutral comments such as that identified in the correct option will avoid negative interpretations. The client should not be told not to worry, that everyone gets depressed once in a while, or that he or she will feel better, because such statements are inappropriate.

A client with a diagnosis of anorexia nervosa, who is in a state of starvation, is in a two-bed room. A newly admitted client will be assigned to this client's room. Which client would be the best choice as a roommate for the client with anorexia nervosa? 1. A client with pneumonia 2. A client undergoing diagnostic tests 3. A client who thrives on managing others 4. A client who could benefit from the client's assistance at mealtime

2. A client undergoing diagnostic tests Rationale: The client undergoing diagnostic tests is an acceptable roommate. The client with anorexia nervosa is most likely experiencing hematological complications, such as leukopenia. Having a roommate with pneumonia would place the client with anorexia nervosa at risk for infection. The client with anorexia nervosa should not be put in a situation in which the client can focus on the nutritional needs of others or be managed by others because this may contribute to sublimation and suppression of personal hunger.

The nurse is caring for a client with a diagnosis of agoraphobia. When communicating with the client about the disorder, the nurse should expect the client to describe which behavior? 1. A fear of dirt and germs 2. A fear of leaving the house 3. A fear of speaking in public 4. A fear of riding in elevators

2. A fear of leaving the house Rationale: Agoraphobia is a fear of leaving the house and experiencing panic attacks when doing so. Option 1 describes an obsessive-compulsive behavior. Option 3 describes a social phobia. Option 4 describes claustrophobia.

The nurse is caring for a client who is experiencing disturbed thought processes as a result of paranoia. In formulating nursing interventions with the members of the health care team, what best instruction should the nurse provide to the staff? 1. Increase socialization of the client with peers. 2. Avoid laughing or whispering in front of the client. 3. Begin to educate the client about social supports in the community. 4. Have the client sign a release of information to appropriate parties for assessment purposes.

2. Avoid laughing or whispering in front of the client. Rationale: Disturbed thought process related to paranoia is the client's problem, and the plan of care must address this problem. The client is experiencing paranoia and is distrustful and suspicious of others. The members of the health care team need to establish a rapport and trust with the client. Laughing or whispering in front of the client would be counterproductive. The remaining options ask the client to trust on a multitude of levels. These options are actions that are too intrusive for a client who is paranoid.

The health care provider has prescribed medication therapy for a client with an alcohol abuse problem to assist in the maintenance of sobriety. The nurse reviews the client's record and expects to note that which medication has been prescribed? 1. Clonidine (Catapres) 2. Disulfiram (Antabuse) 3. Pyridoxine hydrochloride (vitamin B6) 4. Chlordiazepoxide hydrochloride (Librium)

2. Disulfiram (Antabuse) Rationale: Disulfiram is a medication used for alcoholism, and it aids in the maintenance of sobriety. Clonidine is an antihypertensive medication. Pyridoxine hydrochloride is used in the treatment of vitamin B6 deficiency. Chlordiazepoxide hydrochloride is an antianxiety medication (a benzodiazepine) that is used in the management of acute alcohol withdrawal symptoms.

A client with anorexia nervosa is a member of a predischarge support group. The client verbalizes that she would like to buy some new clothes, but her finances are limited. Group members have brought some used clothes to the client to replace the client's old clothes. The client believes that the new clothes were much too tight and has reduced her calorie intake to 800 calories daily. How should the nurse evaluate this behavior? 1. Normal behavior 2. Evidence of the client's disturbed body image 3. Regression as the client is moving toward the community 4. Indicative of the client's ambivalence about hospital discharge

2. Evidence of the client's disturbed body image Rationale: Disturbed body image is a concern with clients with anorexia nervosa. Although the client may struggle with ambivalence and show regressed behavior, the client's coping pattern relates to the basic issue of disturbed body image. The nurse should address this need in the support group.

The police arrive at the emergency department with a client who has lacerated both wrists. What is the initial nursing action? 1. Administer an antianxiety agent. 2. Examine and treat the wound sites. 3. Secure and record a detailed history. 4. Encourage and assist the client to ventilate feelings.

2. Examine and treat the wound sites. Rationale: The initial nursing action is to assess and treat the self-inflicted injuries. Injuries from lacerated wrists can lead to a life-threatening situation. Other interventions, such as options 1, 3, and 4, may follow after the client has been treated medically.

When planning the discharge of a client with chronic anxiety, the nurse directs the goals at promoting a safe environment at home. Which is the most appropriate maintenance goal? 1. Suppressing feelings of anxiety 2. Identifying anxiety-producing situations 3. Continued contact with a crisis counselor 4. Eliminating all anxiety from daily situations

2. Identifying anxiety-producing situations Rationale: Recognizing situations that produce anxiety allows the client to prepare to cope with anxiety or avoid a specific stimulus. Counselors will not be available for all anxiety-producing situations, and this option does not encourage the development of internal strengths. Suppressing feelings will not resolve anxiety. Elimination of all anxiety from life is impossible.

The mental health nurse is conducting a group therapy session and is monitoring a client with a diagnosis of agoraphobia who has been attending the sessions for several months. The nurse notes that the client is cooperative, sharing with peers, and making appropriate suggestions during group discussions. How should the nurse interpret this behavior? 1. Manipulation 2. Improvement 3. Attention seeking 4. Desire to be accepted

2. Improvement Rationale: The behaviors identified in the question indicate improvement in the client's condition. The question presents no information indicating that the client is being manipulative. Acting out is attention-seeking behavior. All clients have a desire to be accepted.

The nurse is caring for a female client who was admitted to the mental health unit recently for anorexia nervosa. The nurse enters the client's room and notes that the client is engaged in rigorous push-ups. Which nursing action is most appropriate? 1. Interrupt the client and weigh her immediately. 2. Interrupt the client and offer to take her for a walk. 3. Allow the client to complete her exercise program. 4. Tell the client that she is not allowed to exercise rigorously.

2. Interrupt the client and offer to take her for a walk. Rationale: Clients with anorexia nervosa frequently are preoccupied with rigorous exercise and push themselves beyond normal limits to work off caloric intake. The nurse must provide for appropriate exercise and place limits on rigorous activities. The correct option stops the harmful behavior yet provides the client with an activity to decrease anxiety that is not harmful. Weighing the client immediately reinforces the client's preoccupation with weight. Allowing the client to complete the exercise program can be harmful to the client. Telling the client that she is not allowed to complete the exercise program will increase the client's anxiety.

The nurse is preparing a client for the termination phase of the nurse-client relationship. The nurse prepares to implement which nursing task that is most appropriate for this phase? 1. Planning short-term goals 2. Making appointment referrals 3. Developing realistic solutions 4. Identifying expected outcomes

2. Making appointment referrals Rationale: Tasks of the termination phase include evaluating client performance, evaluating achievement of expected outcomes, evaluating future needs, making appropriate referrals, and dealing with the common behaviors associated with termination. The remaining options identify tasks appropriate for the working phase of the relationship.

The nurse in the emergency department is caring for a young female victim of sexual assault. The client's physical assessment is complete, and physical evidence has been collected. The nurse notes that the client is withdrawn, confused, and at times physically immobile. How should the nurse interpret these behaviors? 1. Signs of depression 2. Normal reactions to a devastating event 3. Evidence that the client is a high suicide risk 4. Indicative of the need for hospital admission

2. Normal reactions to a devastating event Rationale: During the acute phase of the rape crisis, the client can display a wide range of emotional and somatic responses. The symptoms noted indicate a normal reaction. Options 1, 3, and 4 are incorrect interpretations.

A client has been diagnosed with major depression. The nurse notes that the client is not eating adequately and at times refuses to eat. What should the nurse plan to do to meet the client's nutritional needs? 1. Force foods and fluids. 2. Provide small, frequent meals. 3. Provide snacks and meals as requested. 4. Tell the client that social activities will be restricted unless food intake is increased.

2. Provide small, frequent meals. Rationale: A depressed client may eat small amounts of food because large amounts may seem overwhelming. If the client becomes overwhelmed, he or she may respond by withdrawing further. Providing snacks and meals when the client requests them will not ensure adequate nutritional intake. Forcing foods and fluids and telling the client that social activities will be restricted will cause further withdrawal by the client. Telling the client that social activities will be restricted also is a demeaning action.

A client is unwilling to go out of the house for fear of "making a fool of myself in public." Because of this fear, the client remains homebound. Based on these data, which mental health disorder is the client experiencing? 1. Agoraphobia 2. Social phobia 3. Claustrophobia 4. Hypochondriasis

2. Social phobia Rationale: Social phobia is a fear of situations in which one might be embarrassed or criticized, such as the fear of speaking, performing, or eating in public. The person fears making a fool of oneself. Agoraphobia is a fear of open spaces and the fear of being trapped in a situation from which there may not be an escape. Claustrophobia is a fear of closed places. Clients with hypochondriacal symptoms focus their anxiety on physical complaints and are preoccupied with their health.

The mental health nurse is reviewing the discharge plan for a hospitalized client. In reviewing the plan, the nurse recognizes that which is the most prominent problem in the management of a client with a mental health problem in the community? 1. The community's opposition 2. The client's noncompliance with medication therapy 3. The associated increased incidence of social problems 4. The family's reaction to keeping the client in the community

2. The client's noncompliance with medication therapy Rationale: Clients often forget to take their medications as scheduled, and this is the most prominent problem. Options 1, 3, and 4 may occur, but the problems described are not the most prominent and can be addressed and often controlled.

The nurse is reviewing the assessment data of a client admitted to the mental health unit. The nurse notes that the admission nurse documented that the client is experiencing anxiety as a result of a situational crisis. The nurse determines that this type of crisis could be caused by which event? 1. Witnessing a murder 2. The death of a loved one 3. A fire that destroyed the client's home 4. A recent rape episode experienced by the client

2. The death of a loved one Rationale: A situational crisis arises from external rather than internal sources. External situations that could precipitate a crisis include loss of or change of a job, the death of a loved one, abortion, change in financial status, divorce, addition of new family members, pregnancy, and severe illness. Options 1, 3, and 4 identify adventitious crises. An adventitious crisis refers to a crisis of disaster; it is unplanned or accidental.

A client diagnosed with delirium becomes disoriented and confused at night. Which intervention should the nurse implement initially? 1. Move the client next to the nurse's station. 2. Use an indirect light source and turn off the television. 3. Keep the television and a soft light on during the night. 4. Play soft music during the night, and maintain a well-lit room.

2. Use an indirect light source and turn off the television. Rationale: Provision of a consistent daily routine and a low stimulating environment is important when a client is disoriented. Noise, including radio and television, may add to the confusion and disorientation. Moving the client next to the nurses' station may become necessary but is not the initial action.

The nurse is planning activities for a client diagnosed with bipolar disorder with aggressive social behavior. Which activity would be most appropriate for this client? 1. Chess 2. Writing 3. Ping Pong 4. Basketball

2. Writing Rationale: Solitary activities that require a short attention span with mild physical exertion are the most appropriate activities for a client who is exhibiting aggressive behavior. Writing (journaling), walks with staff, and finger painting are activities that minimize stimuli and provide a constructive release for tension. The remaining options have a competitive element to them and should be avoided because they can stimulate aggression and increase psychomotor activity.

A client says to the nurse, "The federal guards were sent to kill me." What is the best nursing response to the client's concern? 1. "I don't believe this is true." 2. "The guards are not out to kill you." 3. "Do you feel afraid that people are trying to hurt you?" 4. "What makes you think the guards were sent to hurt you?"

3. "Do you feel afraid that people are trying to hurt you?" Rationale: It is most therapeutic for the nurse to empathize with the client's experience. The remaining options lack this connection with the client. Disagreeing with delusions may make the client more defensive, and the client may cling to the delusions even more. Encouraging discussion regarding the delusion is inappropriate.

A female victim of a sexual assault is being seen in the crisis center. The client states that she still feels "as though the rape just happened yesterday," even though it has been a few months since the incident. What is the most appropriate nursing response? 1. "You need to try to be realistic. The rape did not just occur." 2. "It will take some time to get over these feelings about your rape." 3. "Tell me more about the incident that causes you to feel like the rape just occurred." 4. "What do you think that you can do to alleviate some of your fears about being raped again?"

3. "Tell me more about the incident that causes you to feel like the rape just occurred." Rationale: The correct option allows the client to express her ideas and feelings more fully and portrays a nonhurried, nonjudgmental, supportive attitude on the part of the nurse. Clients need to be reassured that their feelings are normal and that they may express their concerns freely in a safe, caring environment. Option 1 immediately blocks communication. Option 2 places the client's feelings on hold. Option 4 places the problem-solving totally on the client.

The nurse is conducting an initial assessment on a client in crisis. When assessing the client's perception of the precipitating event that led to the crisis, what is the most appropriate question? 1. "With whom do you live?" 2. "Who is available to help you?" 3. "What leads you to seek help now?" 4. "What do you usually do to feel better?"

3. "What leads you to seek help now?" Rationale: The nurse's initial task when assessing a client in crisis is to assess the individual or family and the problem. The more clearly the problem can be defined, the better the chance a solution can be found. The correct option would assist in determining data related to the precipitating event that led to the crisis. Options 1 and 2 assess situational supports. Option 4 assesses personal coping skills.

The nurse observes that a client with a potential for violence is agitated, pacing up and down the hallway, and is making aggressive and belligerent gestures at other clients. Which statement would be most appropriate to make to this client? 1. "You need to stop that behavior now." 2. "You will need to be placed in seclusion." 3. "You seem restless; tell me what is happening." 4. "You will need to be restrained if you do not change your behavior."

3. "You seem restless; tell me what is happening." Rationale: The best statement is to ask the client what is causing the agitation. This will assist the client to become aware of the behavior and may assist the nurse in planning appropriate interventions for the client. Option 1 is demanding behavior that could cause increased agitation in the client. Options 2 and 4 are threats to the client and are inappropriate.

A client diagnosed with terminal cancer says to the nurse, "I'm going to die, and I wish my family would stop hoping for a cure! I get so angry when they carry on like this. After all, I'm the one who's dying." Which response by the nurse is therapeutic? 1. "Have you shared your feelings with your family?" 2. "I think we should talk more about your anger with your family." 3. "You're feeling angry that your family continues to hope for you to be cured?" 4. "You are probably very depressed, which is understandable with such a diagnosis."

3. "You're feeling angry that your family continues to hope for you to be cured?" Rationale: Restating is a therapeutic communication technique in which the nurse repeats what the client says to show understanding and to review what was said. While it is appropriate for the nurse to attempt to assess the client's ability to discuss feelings openly with family members, it does not help the client discuss the feelings causing the anger. The nurse's attempt to focus on the central issue of anger is premature. The nurse would never make a judgment regarding the reason for the client's feeling; this is nontherapeutic in the one-to-one relationship.

When the mental health nurse visits a client at home, the client states, "I haven't slept at all the last couple of nights." Which response by the nurse illustrates a therapeutic communication response to this client? 1. "I see." 2. "Really?" 3. "You're having difficulty sleeping?" 4. "Sometimes, I have trouble sleeping too."

3. "You're having difficulty sleeping?" Rationale: The correct option uses the therapeutic communication technique of restatement. Although restatement is a technique that has a prompting component to it, it repeats the client's major theme, which assists the nurse to obtain a more specific perception of the problem from the client. The remaining options are not therapeutic responses since none encourage the client to expand on the problem. Offering personal experiences moves the focus away from the client and onto the nurse.

Which describes the primary focus of milieu therapy? 1. A form of behavior modification therapy 2. A cognitive approach to changing behavior 3. A living, learning, or working environment 4. A behavioral approach to changing behavior

3. A living, learning, or working environment Rationale: Milieu therapy, or "therapeutic community," has as its focus a living, learning, or working environment. Such therapy may be based on numerous therapeutic modalities ranging from structured behavioral therapy to spontaneous, humanistically oriented approaches. Although milieu therapy may include behavioral approaches, the correct option describes its primary focus.

The nurse employed in a mental health unit of a hospital is the leader of a group psychotherapy session. What is the nurse's role during the termination stage of group development? 1. Acknowledging that the group has identified goals 2. Encouraging the accomplishment of the group's work 3. Acknowledging the contributions of each group member 4. Encouraging members to become acquainted with one another

3. Acknowledging the contributions of each group member Rationale: In the termination stage, the group leader's task is to acknowledge the contributions of each member and the experience of the group as a whole. In this stage, the group members prepare for separation and assist each other to prepare for the future. Acknowledging that the group has identified goals and encouraging group bonding both occur during the initial stage. Encouraging accomplishment of the group's work is appropriate during the working stage.

A client is admitted to the mental health unit after an attempted suicide by hanging. The nurse can best ensure client safety by which action? 1. Requesting that a peer remain with the client at all times 2. Removing the client's clothing and placing the client in a hospital gown 3. Assigning a staff member to the client who will remain with the client at all times 4. Admitting the client to a seclusion room where all potentially dangerous articles are removed

3. Assigning a staff member to the client who will remain with the client at all times Rationale: Hanging is a serious suicide attempt. The plan of care must reflect action that ensures the client's safety. Constant observation status (one-to-one) with a staff member is the best choice. Placing the client in a hospital gown and requesting that a peer remain with the client would not ensure a safe environment. Seclusion should not be the initial intervention, and the least restrictive measure should be used.

A client is admitted to a medical nursing unit with a diagnosis of acute blindness after being involved in a hit-and-run accident. When diagnostic testing cannot identify any organic reason why this client cannot see, a mental health consult is prescribed. Which condition will be the focus of this consult? 1. Psychosis 2. Repression 3. Conversion disorder 4. Dissociative disorder

3. Conversion disorder Rationale: A conversion disorder is the alteration or loss of a physical function that cannot be explained by any known pathophysiological mechanism. A conversion disorder is thought to be an expression of a psychological need or conflict. In this situation, the client witnessed an accident that was so psychologically painful that the client became blind. Psychosis is a state in which a person's mental capacity to recognize reality, communicate, and relate to others is impaired, interfering with the person's ability to deal with life's demands. Repression is a coping mechanism in which unacceptable feelings are kept out of awareness. A dissociative disorder is a disturbance or alteration in the normally integrative functions of identity, memory, or consciousness.

A manic client begins to make sexual advance towards visitors in the dayroom. When the nurse firmly states that this is inappropriate and will not be allowed, the client becomes verbally abusive and threatens physical violence to the nurse. Based on the analysis of this situation, which intervention should the nurse implement? 1. Place the client in seclusion for 30 minutes. 2. Tell the client that the behavior is inappropriate. 3. Escort the client to their room, with the assistance of other staff. 4. Tell the client that their telephone privileges are revoked for 24 hours.

3. Escort the client to their room, with the assistance of other staff. Rationale: The client is at risk for injury to self and others and should be escorted out of the dayroom. Seclusion is premature in this situation. Telling the client that the behavior is inappropriate has already been attempted by the nurse. Denying privileges may increase the agitation that already exists in this client.

A depressed client verbalizes feelings of low self-esteem and self-worth typified by statements such as "I'm such a failure. I can't do anything right." How should the nurse plan on responding to the client's statement? 1. Reassure the client that things will get better. 2. Tell the client that this is not true and that we all have a purpose in life. 3. Identify recent behaviors or accomplishments that demonstrate the client's skills. 4. Remain with the client and sit in silence; this will encourage the client to verbalize feelings.

3. Identify recent behaviors or accomplishments that demonstrate the client's skills. Rationale: Feelings of low self-esteem and worthlessness are common symptoms of a depressed client. An effective plan of care to enhance the client's personal self-esteem is to provide experiences for the client that are challenging, but that will not be met with failure. Reminders of the client's past accomplishments or personal successes are ways to interrupt the client's negative self-talk and distorted cognitive view of self. Options 1 and 2 give advice and devalue the client's feelings. Silence may be interpreted as agreement.

A moderately depressed client who was hospitalized 2 days ago suddenly begins smiling and reporting that the crisis is over. The client says to the nurse, "I'm finally cured." How should the nurse interpret this behavior as a cue to modify the treatment plan? 1. Suggesting a reduction of medication 2. Allowing increased "in-room" activities 3. Increasing the level of suicide precautions 4. Allowing the client off-unit privileges as needed

3. Increasing the level of suicide precautions Rationale: A client who is moderately depressed and has only been in the hospital 2 days is unlikely to have such a dramatic cure. When a depression suddenly lifts, it is likely that the client may have made the decision to harm himself or herself. Suicide precautions are necessary to keep the client safe. The remaining options are therefore incorrect interpretations.

The nurse is developing a plan of care for a client who was experiencing anxiety after the loss of a job. The client is now verbalizing concerns regarding the ability to meet role expectations and financial obligations. What is the priority problem for this client? 1. Anxiety 2. Unrealistic outlook 3. Lack of ability to cope effectively 4. Disturbances in thoughts and ideas

3. Lack of ability to cope effectively Rationale: Lack of ability to cope effectively may be evidenced by a client's inability to meet basic needs, inability to meet role expectations, alteration in social participation, use of inappropriate defense mechanisms, or impairment of usual patterns of communication. Anxiety is a broad description and can occur as a result of many triggers and although the client was experiencing anxiety, the client's concern now is the ability to meet role expectations and financial obligations. There is no information in the question that indicates an unrealistic outlook or disturbances in thoughts and ideas.

The nurse is caring for a client with anorexia nervosa. Which behavior is characteristic of this disorder and reflects anxiety management? 1. Engaging in immoral acts 2. Always reinforcing self-approval 3. Observing rigid rules and regulations 4. Having the need always to make the right decision

3. Observing rigid rules and regulations Rationale: Clients with anorexia nervosa have the desire to please others. Their need to be correct or perfect interferes with rational decision-making processes. These clients are moralistic. Rules and rituals help these clients manage their anxiety.

A client comes to the emergency department after an assault and is extremely agitated, trembling, and hyperventilating. What is the priority nursing action for this client? 1. Begin to teach relaxation techniques. 2. Encourage the client to discuss the assault. 3. Remain with the client until the anxiety decreases. 4. Place the client in a quiet room alone to decrease stimulation.

3. Remain with the client until the anxiety decreases. Rationale: This client is in a severe state of anxiety. When a client is in a severe or panic state of anxiety, it is crucial for the nurse to remain with the client. The client in a severe state of anxiety would be unable to learn relaxation techniques. Discussing the assault at this point would increase the client's level of anxiety further. Placing the client in a quiet room alone may also increase the anxiety level.

The emergency department nurse is caring for a client who has been identified as a victim of physical abuse. In planning care for the client, which is the priority nursing action? 1. Adhering to the mandatory abuse-reporting laws 2. Notifying the case worker of the family situation 3. Removing the client from any immediate danger 4. Obtaining treatment for the abusing family member

3. Removing the client from any immediate danger Rationale: Whenever an abused client remains in the abusive environment, priority must be placed on ascertaining whether the client is in any immediate danger. If so, emergency action must be taken to remove the client from the abusing situation. Options 1, 2, and 4 may be appropriate interventions, but are not the priority.

A client is being prepared for electroconvulsive therapy (ECT). The nurse's plan of care for the day before ECT includes ensuring that the client follows which guideline? 1. Does not smoke at all 2. Receives no visitors and participates in limited unit activities 3. Reports to the clinic for blood draws and an electrocardiogram (ECG) 4. Is placed on nothing by mouth (NPO) status for 16 to 24 hours before the ECT

3. Reports to the clinic for blood draws and an electrocardiogram (ECG) Rationale: Before ECT, blood tests are performed and an ECG is done to determine a baseline status of the client. The nurse needs to explain the need for these preprocedures to the client. Maintaining *NPO status for 6 to 8 hours before treatment* is adequate; NPO status for 16 to 24 hours is not necessary. Some hospitals place clients on NPO status at midnight before ECT in the morning. Some clients who are on cardiovascular medication may be instructed to take their medicine with sips of water several hours before ECT. Options 1 and 2 are incorrect.

The nurse is caring for a client diagnosed with catatonic stupor who is lying on the bed in a fetal position. What is the most appropriate nursing intervention? 1. Ask direct questions to encourage talking. 2. Leave the client alone so as to minimize external stimuli. 3. Sit beside the client in silence with occasional open-ended questions. 4. Take the client into the dayroom with other clients so that they can help watch him.

3. Sit beside the client in silence with occasional open-ended questions. Rationale: Clients who are withdrawn may be immobile and mute and may require consistent, repeated approaches. Communication with withdrawn clients requires much patience from the nurse. Interventions include the establishment of interpersonal contact. The nurse facilitates communication with the client by sitting in silence, asking open-ended questions rather than direct questions, and pausing to provide opportunities for the client to respond. While overstimulation is not appropriate, there is no therapeutic value in ignoring the client. The client's safety is not the responsibility of other clients.

The nurse in the mental health unit is performing an assessment in a client who has a history of multiple somatic complaints involving several organ systems. Diagnostic studies revealed no organic pathology. The care plan developed for this client will reflect that the client is experiencing which disorder? 1. Depression 2. Schizophrenia 3. Somatization disorder 4. Obsessive-compulsive disorder

3. Somatization disorder Rationale: Somatization disorder is characterized by a long history of multiple physical problems with no satisfactory organic explanation. The clinical findings associated with schizophrenia, depression, and obsessive-compulsive disorder are unrelated to somatic complaints.

A nursing instructor teaches a group of nursing students about violence in the family. Which statement by a student indicates a need for further teaching? 1. "Abusers use fear and intimidation." 2. "Abusers usually have poor self-esteem." 3. "Abusers often are jealous or self-centered." 4. "Abuse occurs more often in low-income families."

4. "Abuse occurs more often in low-income families." Rationale: Personal characteristics of abusers include low self-esteem, immaturity, dependence, insecurity, and jealousy. Abusers often use fear and intimidation to the point at which their victims will do anything just to avoid further abuse. The statement that abuse occurs more often in lower socioeconomic groups is incorrect.

A client with a diagnosis of depression has been meeting with the mental health nurse for therapy sessions for the past 6 weeks. During the session the client says to the nurse, "I lost my job this week, and I'm going to be evicted from my apartment if I can't pay my bill. The only person that I have is my daughter, but I don't want to burden her with my problems." Which response by the nurse would be therapeutic? 1. "Why did you lose your job?" 2. "There are homeless shelters available, and we will get you into one if you are evicted from your apartment." 3. "If you get evicted from your apartment, we will commit you to the hospital, so you will have a place to eat and sleep." 4. "Let's talk about contacting your daughter. Wouldn't you want to know if your daughter was having difficulty and try to help her if you could?"

4. "Let's talk about contacting your daughter. Wouldn't you want to know if your daughter was having difficulty and try to help her if you could?" Rationale: The therapeutic communication technique is clarification that attempts to put vague ideas into words. It helps the client to view the explicit correlation between the client's feelings and actions. Asking why a client lost a job is not directly related to the client's feelings and concerns. Offering to provide a homeless shelter or to commit the client to the hospital does not address the issue at hand and places the client's concerns and feelings on hold.

A client experiencing a great deal of stress and anxiety is being taught to use self-control therapy. Which statement by the client indicates a need for further teaching about the therapy? 1. "This form of therapy can be applied to new situations." 2. "An advantage of this technique is that change is likely to last." 3. "Talking to oneself is a basic component of this form of therapy." 4. "This form of therapy provides a negative reinforcement when the stimulus is produced."

4. "This form of therapy provides a negative reinforcement when the stimulus is produced." Rationale: Negative reinforcement when the stimulus is produced is descriptive of aversion therapy. Options 1, 2, and 3 are characteristics of self-control therapy.

During a therapy session with a client with paranoid disorder, the client says to the nurse, "You look so nice today. I love how you do your hair, and I love that perfume you're wearing." Which response by the nurse would be therapeutic? 1. "Your comment is inappropriate." 2. "Thank you for noticing. I just bought this new perfume." 3. "My hair has been a mess. I really needed to have it done." 4. "We are not here to discuss how I look or smell. We are here to talk about you."

4. "We are not here to discuss how I look or smell. We are here to talk about you." Rationale: The therapeutic response by the nurse is the one that clarifies the content of the client's statements and directs the client to the purpose of the session. The nurse should confront the client verbally regarding the inappropriate statements and refocus the client back to the issue of the session. Option 1 may be judgmental and may provide an opening for a verbal struggle. Options 2 and 3 are social responses and could be misinterpreted by the client.

The nurse is preparing a client with a history of command hallucinations for discharge by providing instructions on interventions for managing hallucinations and anxiety. Which statement in response to these instructions suggests to the nurse that the client understands the instructions? 1. "My medications aren't likely to make me anxious." 2. "I'll go to support group and talk so that I don't hurt anyone." 3. "It's not likely that I'll get anxious or hear things if I get enough sleep and eat well." 4. "When I begin to hallucinate, I'll call my therapist and talk about what I should do."

4. "When I begin to hallucinate, I'll call my therapist and talk about what I should do." Rationale: The risk for impulsive and aggressive behavior may increase if a client is receiving command hallucinations to harm self or others. If the client is experiencing a hallucination, the nurse should ask the client whether he or she has intentions to hurt him- or herself or others. Talking about auditory hallucinations can interfere with subvocal muscular activity associated with a hallucination. The client statements in the remaining options will aid in wellness, but are not specific interventions for hallucinations, if they occur.

The mental health nurse is caring for a client with a social phobia. The nurse tells the client that a music therapy session has been scheduled as part of the treatment plan. The client tells the nurse that she cannot sing and refuses to attend. What is the appropriate nursing response? 1. "You must go. You have no choice." 2. "Why don't you want to attend? What is the real reason?" 3. "The health care provider has prescribed this therapy for you." 4. "You don't have to sing at the session. You can listen and enjoy the music."

4. "You don't have to sing at the session. You can listen and enjoy the music." Rationale: The correct option encourages the client to socialize and indicates that it is not necessary to sing. Option 2 asks why, and use of this word should be avoided. Options 1 and 3 imply a demand and do not address the client's concern. The correct option is the only one that addresses the client's concern.

A depressed client on an inpatient unit says to the nurse, "My family would be better off without me." What is the nurse's best response? 1. "Have you talked to your family about this?" 2. "Everyone feels this way when they are depressed." 3. "You will feel better once your medication begins to work." 4. "You sound very upset. Are you thinking of hurting yourself?"

4. "You sound very upset. Are you thinking of hurting yourself?" Rationale: Clients who are depressed may be at risk for suicide. It is critical for the nurse to assess suicidal ideation and plan. The nurse should ask the client directly whether a plan for self-harm exists. Options 1, 2, and 3 do not deal directly with the client's feelings.

A client with a diagnosis of major depression who has attempted suicide says to the nurse, "I should have died. I've always been a failure. Nothing ever goes right for me." Which response demonstrates therapeutic communication? 1. "You have everything to live for." 2. "Why do you see yourself as a failure?" 3. "Feeling like this is all part of being depressed." 4. "You've been feeling like a failure for a while?"

4. "You've been feeling like a failure for a while?" Rationale: Responding to the feelings expressed by a client is an effective therapeutic communication technique. The correct option is an example of the use of restating. The remaining options block communication because they minimize the client's experience and do not facilitate exploration of the client's expressed feelings. In addition, use of the word "why" is nontherapeutic.

The nurse is developing a plan of care for a client in a crisis state. When developing the plan, the nurse should consider which factor? 1. A crisis state indicates that the client has a mental illness. 2. A crisis state indicates that the client has an emotional illness. 3. Presenting symptoms in a crisis situation are similar for all clients experiencing a crisis. 4. A client's response to a crisis is individualized and what constitutes a crisis for one client may not constitute a crisis for another client.

4. A client's response to a crisis is individualized and what constitutes a crisis for one client may not constitute a crisis for another client. Rationale: Although each crisis response can be described in similar terms as far as presenting symptoms are concerned, what constitutes a crisis for one client may not constitute a crisis for another client because each is a unique individual. Being in the crisis state does not mean that the client has a mental or emotional illness.

A client is admitted to the mental health unit with a diagnosis of depression. The nurse should develop a plan of care for the client that includes which intervention? 1. Encouraging quiet reading and writing for the first few days 2. Identification of physical activities that will provide exercise 3. No socializing activities, until the client asks to participate in milieu 4. A structured program of activities in which the client can participate

4. A structured program of activities in which the client can participate Rationale: A client with depression often is withdrawn while experiencing difficulty concentrating, loss of interest or pleasure, low energy, fatigue, and feelings of worthlessness and poor self-esteem. The plan of care needs to provide successful experiences in a stimulating yet structured environment. The remaining options are either too "restrictive" or offer little or no structure and stimulation.

On review of the client's record, the nurse notes that the mental health admission was voluntary. Based on this information, the nurse anticipates which client behavior? 1. Fearfulness regarding treatment measures. 2. Anger and aggressiveness directed toward others. 3. An understanding of the pathology and symptoms of the diagnosis. 4. A willingness to participate in the planning of the care and treatment plan.

4. A willingness to participate in the planning of the care and treatment plan. Rationale: In general, clients seek voluntary admission. If a client seeks voluntary admission, the most likely expectation is that the client will participate in the treatment program since they are actively seeking help. The remaining options are not characteristics of this type of admission. Fearfulness, anger, and aggressiveness are more characteristic of an involuntary admission. Voluntary admission does not guarantee a client's understanding of their illness, only of their desire for help.

The nurse is admitting a client with a diagnosis of posttraumatic stress disorder to the mental health unit. The client is confused and disoriented. During the assessment, what is the nurse's primary goal for this client? 1. Explain the unit rules. 2. Orient the client to the unit. 3. Stabilize the client's psychiatric needs. 4. Accept the client and make the client feel safe.

4. Accept the client and make the client feel safe. Rationale: It is important to make a confused client feel safe. Explaining the unit rules and orientation to the unit are part of any admission process. Stabilizing psychiatric needs is a long-term goal.

During a home visit, the nurse suspects that a young daughter of the client is bulimic. The nurse bases this suspicion on which primary characteristic of bulimia? 1. Refusing to eat and excessive exercising 2. Eating only vegetables and fruits and fasting 3. Hoarding of food and difficulty controlling food intake 4. Eating a lot of food in a short period of time and misuse of laxatives

4. Eating a lot of food in a short period of time and misuse of laxatives Rationale: Eating binges and purging are the characteristic that would be seen in bulimia. Eating only certain types of foods may reflect a preference but does not indicate bulimia. Bulimic persons usually do not refuse to eat; rather, they binge and purge. Hoarding of food may indicate another problem.

A client recently admitted to the hospital in the manic phase of bipolar disorder is dehydrated, unkempt, taking antipsychotic medications, and complaining of abdominal fullness and discomfort. The nurse determines that which intervention is most appropriate for these complaints? 1. Teach self-grooming skills. 2. Reward cleanliness with unit privileges. 3. Monitor the adequacy of the antipsychotic dosage. 4. Encourage frequent fluid intake and a high-fiber diet.

4. Encourage frequent fluid intake and a high-fiber diet. Rationale: Constipation is a common elimination problem with clients in a manic phase of bipolar disorder. Constipation may occur as the result of a combination of factors, including taking antipsychotic medications, suppressing the urge to defecate, and a decreased fluid intake as a result of the manic activity level. The symptoms listed in the question, dehydrated, unkempt, and abdominal fullness and discomfort, in combination with antipsychotic medications, are indicators of constipation. A high-fiber diet and increased fluids can reduce constipation.

When a client is admitted to an inpatient mental health unit with the diagnosis of anorexia nervosa, a cognitive behavioral approach is used as part of the treatment plan. The nurse understands that which is the purpose of this approach? 1. Providing a supportive environment 2. Examining intrapsychic conflicts and past issues 3. Emphasizing social interaction with clients who withdraw 4. Helping the client to examine dysfunctional thoughts and beliefs

4. Helping the client to examine dysfunctional thoughts and beliefs Rationale: Cognitive behavioral therapy is used to help the client identify and examine dysfunctional thoughts and to identify and examine values and beliefs that maintain these thoughts. The remaining options, while therapeutic in certain situations, are not the focus of cognitive behavioral therapy.

The nurse is monitoring a hospitalized client who abuses alcohol. Which findings should alert the nurse to the potential for alcohol withdrawal delirium? 1. Hypotension, ataxia, hunger 2. Stupor, lethargy, muscular rigidity 3. Hypotension, coarse hand tremors, lethargy 4. Hypertension, changes in level of consciousness, hallucinations

4. Hypertension, changes in level of consciousness, hallucinations Rationale: Symptoms associated with alcohol withdrawal delirium typically include anxiety, insomnia, anorexia, hypertension, disorientation, hallucinations, changes in level of consciousness, agitation, fever, and delusions.

The nurse is monitoring a client with a diagnosis of schizophrenia. The nurse notes that the client's emotional responses to situations occurring throughout the day are incongruent with the tone of the situation. The nurse should document the findings using which description of the client's behavioral response? 1. Flat affect 2. Bizarre affect 3. Blunted affect 4. Inappropriate affect

4. Inappropriate affect Rationale: An inappropriate affect refers to an emotional response to a situation that is incongruent with the tone of the situation. A flat affect is manifested as an immobile facial expression or blank look. A bizarre affect such as grimacing, laughing, and self-directed mumbling is marked when the client is unable to relate logically to the environment. A blunted affect is a minimal emotional response or outward affect that typically does not coincide with the client's inner emotions.

The nurse is planning care for a client with bipolar disorder who is experiencing psychomotor agitation. Which activity should the nurse plan for this client? 1. Reading letters and books in a quiet environment 2. Providing an activity such as checkers for the client 3. Involving the client in a card game with other clients on the unit 4. Including the client in a clay-molding class that is scheduled for today

4. Including the client in a clay-molding class that is scheduled for today Rationale: When a client is experiencing psychomotor agitation, it is best to provide activities that involve the use of hands and gross motor movements. Such activities can include volleyball, finger-painting, drawing, and working with clay. These activities provide an appropriate way for the client to discharge motor tension. Reading and simple card games are sedentary activities. Playing checkers requires concentration and more intensive use of thought processes.

The nurse is working with a client who despite making a heroic effort was unable to rescue a neighbor trapped in a house fire. Which client-focused action should the nurse engage in during the working phase of the nurse-client relationship? 1. Exploring the client's ability to function 2. Exploring the client's potential for self-harm 3. Inquiring about the client's perception or appraisal of why the rescue was unsuccessful 4. Inquiring about and examining the client's feelings for any that may block adaptive coping

4. Inquiring about and examining the client's feelings for any that may block adaptive coping Rational: The client must first deal with feelings and negative responses before the client can work through the meaning of the crisis. The correct option pertains directly to the client's feelings and is client-focused. The remaining options do not directly focus on or address the client's feelings.

The nurse is monitoring a client who has been placed in restraints because of violent behavior. When should the nurse determine that it will be safe to remove the restraints? 1. Administered medication has taken effect. 2. The client verbalizes the reasons for the violent behavior. 3. The client apologizes and tells the nurse that it will never happen again. 4. No acts of aggression have been observed within 1 hour after the release of two of the extremity restraints.

4. No acts of aggression have been observed within 1 hour after the release of two of the extremity restraints. Rationale: The best indicator that the behavior is controlled is the fact that the client exhibits no signs of aggression after partial release of restraints. Options 1, 2, and 3 do not ensure that the client has controlled the behavior.

The nurse assesses a client with the admitting diagnosis of bipolar affective disorder, mania. Which client symptoms require the nurse's immediate action? 1. Incessant talking and sexual innuendoes 2. Grandiose delusions and poor concentration 3. Outlandish behaviors and inappropriate dress 4. Nonstop physical activity and poor nutritional intake

4. Nonstop physical activity and poor nutritional intake Rationale: Mania is a mood characterized by excitement, euphoria, hyperactivity, excessive energy, decreased need for sleep, and impaired ability to concentrate or complete a single train of thought. The client's mood is predominantly elevated, expansive, or irritable. All of the options reflect a client's possible symptoms. The correct option clearly presents a problem, however, that compromises physiological integrity and needs to be addressed immediately.

The nurse has developed a plan of care for a client diagnosed with anorexia nervosa. Which client problem would the nurse select as the priority in the plan of care? 1. Disrupted appearance because of weight 2. Inability to feed self because of weakness 3. Pain because of an inflamed gastric mucosa 4. Nutritional imbalance because of lack of intake

4. Nutritional imbalance because of lack of intake Rationale: The priority client problem for the client with anorexia nervosa is lack of intake and nutritional imbalance. Although the problems identified in options 1, 2, and 3 may be considerations in the plan of care for the client with anorexia nervosa, nutritional imbalance is the priority.

The nurse is reviewing the record of a client admitted to the mental health unit. The nurse notes documentation that the client experiences flashbacks. What diagnosis should the nurse expect to be documented for this client? 1. Anxiety 2. Agoraphobia 3. Schizophrenia 4. Posttraumatic stress disorder (PTSD)

4. Posttraumatic stress disorder (PTSD) Rationale: The major clinical manifestation associated with PTSD is client experience of flashbacks. Flashbacks are not specifically associated with anxiety, agoraphobia, or schizophrenia.

The nurse is planning a stress management seminar for clients in an ambulatory care setting. Which concept should the nurse plan to include in the content of the seminar? 1. Biofeedback has the advantage of using no equipment at all. 2. Guided imagery is a helpful technique but requires video equipment for its use. 3. Confrontation is a useful method for solving potentially stressful conflicts with others. 4. Progressive muscle relaxation techniques are useful for easing tension from many causes.

4. Progressive muscle relaxation techniques are useful for easing tension from many causes. Rationale: Biofeedback, guided imagery, progressive muscle relaxation, and meditation are techniques that the nurse can teach the client to reduce the physical impact of stress on the body and promote a feeling of self-control. Biofeedback uses electronic equipment, whereas each of the other techniques requires no equipment after it is learned. Confrontation is not a stress management technique; it is a communication technique.

The mental health nurse is talking to a client who has been diagnosed with posttraumatic stress disorder. During the conversation, the nurse notes that the client is exhibiting a paranoid stare and that he begins to pace and fidget. What is the appropriate nursing intervention? 1. Allow the client to pace. 2. Escort the client to a quiet room. 3. Change the conversation to a less threatening subject. 4. Share the observation with the client and help the client to recognize his feelings.

4. Share the observation with the client and help the client to recognize his feelings. Rationale: Sharing observations with the client may help him recognize and acknowledge feelings. Allowing the client to pace may also allow him to get out of control. Moving to a quiet room or changing the subject will not help the client to recognize his behaviors and feelings.

While being treated, a client is introduced to short periods of exposure to the phobic object while in a relaxed state. What term is used to describe this form of behavior modification? 1. Milieu therapy 2. Aversion therapy 3. Self-control therapy 4. Systematic desensitization

4. Systematic desensitization Rationale: Systematic desensitization is a form of therapy used when the client is introduced to short periods of exposure to the phobic object while in a relaxed state. Exposure is gradually increased until the anxiety about or fear of the object or situation has ceased. Milieu management refers to providing a safe, therapeutic environment and is applicable to not just this scenario. The remaining options are incorrect since they do not involve the intervention described.

A client's unresolved feelings related to loss would be most likely observed during which phase of the therapeutic nurse-client relationship? 1. Trusting 2. Working 3. Orientation 4. Termination

4. Termination In the termination phase, the relationship comes to a close. Ending treatment sometimes may be traumatic for clients who have come to value the relationship and the help. Because loss is an issue, any unresolved feelings related to loss may resurface during this phase. The remaining options are not specifically associated with this issue of unresolved feelings.

What is the most appropriate nursing action to help manage a manic client who is monopolizing a group therapy session? 1. Ask the client to leave the group for this session only. 2. Refer the client to another group that includes other manic clients. 3. Tell the client to stop monopolizing in a firm but compassionate manner. 4. Thank the client for the input, but inform the client that now others need a chance to contribute.

4. Thank the client for the input, but inform the client that now others need a chance to contribute. Rationale: If a client is monopolizing the group, the nurse must be direct and decisive. The best action is to thank the client and suggest that the client stop talking and try listening to others. Although telling the client to stop monopolizing in a firm but compassionate manner may be a direct response, the correct option is more specific and provides direction for the client. The remaining options are inappropriate since they are not directed towards helping the client in a therapeutic manner.

A homebound client confidentially discusses suicidal plans with the visiting nurse. Based on professional duty to observe confidentiality, which statement best describes the nurse's obligation to the client? 1. The nurse must have the client go to the local mental health center daily for counseling. 2. The nurse must ask the client not to reveal suicidal plans if the information needs to be kept confidential. 3. The nurse cannot tell anyone what the client said and must strictly adhere to the professional duty for confidentiality. 4. The nurse must override the duty to observe confidentiality and notify the client's health care provider (HCP) about the suicidal ideation.

4. The nurse must override the duty to observe confidentiality and notify the client's health care provider (HCP) about the suicidal ideation. Rationale: In this situation, the nurse must override the duty to observe confidentiality and notify the client's HCP about the client's suicidal ideation. Option 1 is incorrect because the client is homebound. Option 2 is incorrect because the nurse has a professional obligation to intervene when a client tells the nurse about ideas or plans to harm himself or herself or others. Option 3 is incorrect because the nurse has a moral obligation to protect the client.

A client has been prescribed disulfiram (Antabuse). Before giving the client the first dose of this medication, what should the psychiatric home health nurse determine? 1. If there is a history of hyperthyroidism 2. When the last full meal was consumed 3. If there is a history of diabetes insipidus 4. When the last alcoholic drink was consumed

4. When the last alcoholic drink was consumed Rationale: Disulfiram is an adjunctive treatment for some clients with chronic alcoholism to assist in maintaining enforced sobriety. Because clients must abstain from alcohol for at least 12 hours before the initial dose, the most important assessment is when the last alcoholic intake was consumed. The medication should be used cautiously in clients with hypothyroidism, diabetes mellitus, epilepsy, cerebral damage, nephritis, and hepatic disease. It is contraindicated in persons with severe heart disease, psychosis, or hypersensitivity to the medication.

A hospitalized client is receiving clozapine (Clozaril) for the treatment of a schizophrenic disorder. The nurse determines that the client may be having an adverse reaction to the medication if abnormalities are noted on which laboratory study? 1. Platelet count 2. Cholesterol level 3. Blood urea nitrogen 4. White blood cell (WBC) count

4. White blood cell (WBC) count Rationale: Clozapine is an antipsychotic medication. Clients taking clozapine can experience hematological adverse effects, including agranulocytosis and mild leukopenia. The WBC count should be assessed before initiation of treatment and should be monitored closely during the use of this medication. The client also should be monitored for signs indicating agranulocytosis, which may include sore throat, malaise, and fever. Options 1, 2, and 3 are incorrect and unrelated to this medication.

23. The nurse is taking a history for a female client who is requesting a routine female exam. Which assessment finding requires follow-up? A) Menstruation onset at age 9. B) Contraceptive method includes condoms only. C) Menstrual cycle occurs every 35 days. D) Black-out after one drink last night on a date.

A "black-out" typically occurs after ingestion of alcohol beverages that the client has no recall of the experiences or one's behavior and is indicative of high blood alcohol levels, but the client's experience of a "black-out" after one drink (D) is suspicious of the client receiving a "date rape" drug (Flunitrazepam) and needs additional follow-up. Although (A and C) occur on the outer ranges of "average," both are within acceptable or "normal" ranges. (B) is an individual preference, but using condoms as the only contraceptive method carries a higher chance of conception. Correct Answer(s): D

62. A young adult male client, diagnosed with paranoid schizophrenia, believes that world is trying poison him. What intervention should the nurse include in this client's plan of care? A) Remind the client that his suspicions are not true. B) Ask one nurse to spend time with the client daily. C) Encourage the client to participate in group activities. D) Assign the client to a room closest to the activity room.

A client with paranoid schizophrenia has difficulty with trust and developing a trusting relationship with one nurse (B) is likely to be therapeutic for this client. (A) is argumentative. Stress increases anxiety, and anxiety increases paranoid ideation; (C) would be too stressful and anxiety-promoting for a client who is experiencing pathological suspicions. (D) also might increase anxiety and stress. Correct Answer(s): B

59. The nurse is preparing to administer phenelzine sulfate (Nardil) to a client on the psychiatric unit. Which complaint related to administration of this drug should the nurse expect this client to make? A) My mouth feels like cotton. B) That stuff gives me indigestion. C) This pill gives me diarrhea. D) My urine looks pink.

A dry mouth (A) is an anticholinergic effect that is an expected side effect of MAO inhibitors such as phenelzine sulfate (Nardil). (B, C, and D) are not expected side effects of this medication. Correct Answer(s): A

63. The nurse should hold the next scheduled dose of a client's haloperidol (Haldol) based on which assessment finding(s)? A) Dizziness when standing. B) Shuffling gait and hand tremors. C) Urinary retention. D) Fever of 102° F.

A fever (D) may indicate neuroleptic malignant syndrome (NMS), a potentially fatal complication of antipsychotics. The healthcare provider should be contacted before administering the next dose of Haldol. (A, B, and C) are all adverse effects of Haldol which can be managed. Correct Answer(s): D

30. A woman arrives in the Emergency Center and tells the nurse she thinks she has been raped. The client is sobbing and expresses disbelief that a rape could happen because the man is her best friend. After acknowledging the client's fear and anxiety, how should the nurse respond? A) "I would be very upset and mad if my best friend did that to me." B) "You must feel betrayed, but maybe you might have led him on?" C) "Rape is not limited to strangers and frequently occurs by someone who is known to the victim." D) "This does not sound like rape. Did you change your mind about having sex after the fact?"

A victim of date rape or acquaintance rape is less prone to recognize what is happening because the incident usually involves persons who know each other and the dynamics are different than rape by a stranger. (C) provides confrontation for the client's denial because the victim frequently knows and trusts the perpetrator. Nurses should not express personal feelings (A) when dealing with victims. Suggesting that the client led on the rapist (B) indicates that the sexual assault was somehow the victim's fault. (D) is judgmental and does not display compassion or establish trust between the nurse and the client. Correct Answer(s): C

57. Over a period of several weeks, one male participant of a socialization group at a community day care center for the elderly monopolizes most of the group's time and interrupts others when they are talking. What is the best action for the nurse to take in this situation? A) Talk to the client outside the group about his behavior during group meetings. B) Remind the client to allow others in the group a chance to talk. C) Allow the group to handle the problem. D) Ask the client to join another group.

After several weeks, the group is in the working phase and the group members should be allowed to determine the direction of the group. The nurse should ignore the client's comments and allow the group to handle the situation (C). A good leader should not have separate meetings with group members (A), as such behavior is manipulative on the part of the leader. (B) is dictatorial and is not in keeping with good leadership skills. (D) is avoiding the problem. Remember, identify what phase the group is in--initial, working, or termination--this will help determine communication style. Correct Answer(s): C

75. A 45-year-old female client is admitted to the psychiatric unit for evaluation. Her husband states that she has been reluctant to leave home for the last six months. The client has not gone to work for a month and has been terminated from her job. She has not left the house since that time. This client is displaying symptoms of what condition? A) Claustrophobia. B) Acrophobia. C) Agoraphobia. D) Post-traumatic stress disorder.

Agoraphobia (C) is the fear of crowds or being in an open place. (A) is the fear of being in closed places. (B) is the fear of high places. Remember, a phobia is an unrealistic fear which is associated with severe anxiety. (D) consists of the development of anxiety symptoms following a life event that is particularly serious and stressful (war, witnessing a child killed, etc.) and is experienced with terror, fear, and helplessness--a phobia is different. Correct Answer(s): C

18. Based on non-compliance with the medication regimen, an adult client with a medical diagnosis of substance abuse and schizophrenia was recently switched from oral fluphenazine HCl (Prolixin) to IM fluphenazine decanoate (Prolixin Decanoate). What is most important to teach the client and family about this change in medication regimen? A) Signs and symptoms of extrapyramidal effects (EPS). B) Information about substance abuse and schizophrenia. C) The effects of alcohol and drug interaction. D) The availability of support groups for those with dual diagnoses.

Alcohol enhances the EPS side effects of Prolixin. The half-life of Prolixin PO is 8 hours, whereas the half-life of the Prolixin Decanoate IM is 2 to 4 weeks. That means the side effects of drinking alcohol are far more severe when the client drinks alcohol after taking the long-acting Prolixin Decanoate IM. (A, B, and D) provide valuable information and should be included in the client/family teaching, but they do not have the priority of (C). Correct Answer(s): C

28. A 19-year-old female client with a diagnosis of anorexia nervosa wants to help serve dinner trays to other clients on a psychiatric unit. What action should the nurse take? A) Encourage the client's participation in unit activities by asking her to pass trays for the rest of the week. B) Provide an additional challenge by asking the client to also help feed the older clients. C) Suggest another way for this client to participate in unit activities. D) Tell the client that hospital policy does not permit her to pass trays.

Anorexics gain pleasure from providing others with food and watching them eat. Such behaviors reinforce their perception of self-control. These clients should not be allowed to plan or prepare food for unit activities and their desires to do so should be redirected (C). (A and B) are contraindicated for a client with anorexia nervosa. (D) avoids addressing the problem and is manipulative in that the nurse is blaming hospital policy for treatment protocol. Correct Answer(s): C

A 38 year old client is admitted with a diagnosis of paranoid schizophrenia. When the lunch tray is brought to the room, the client refuses to eat and tells the nurse, "I know you are trying to poison me with that food." Which response by the nurse is the most therapuetic? A. "I'll leave your try her. I am available if you need anything else." B. "You're not being poisoned. Why do you think someone is trying to poison you?" C. "No one on this unt has ever died from poisoning. You are safe her." D. I will talk to your healthcare provider about the possibility of changing your diet."

Answer: A Rationale: (A) is the best choice because the nurse does not argue with the client or demand that the client eat but offers support by agreeing to be there if needed, which provides open, rather than closed, response to the client's statement. (B and C) are challenging the client's delusions, and (B) asks why. Probing questions, which start with why, are usually not therapuetic communication for a psychotic client. (D) has not addressed the actual problem- that is, the client's delusion.

The nurse is caring for a client who is taking the mood stabilizer divalproex sodium (Depakote). Which laboratory finding is most important to include in this client's record? A. Liver function test results B. Creatinine clearance C. Complete blood count D. Chemistry panel

Answer: A Rationale: Depakote is metabolized by the liver and cause hepatotoxicity, so lab findings of liver function tests (A) should be included in the client's record. (B) should be in the client record of those who are receiving lithium because it is excreted by the kidneys. (C and D) are routine lab tests and are not specifically related to the administration of Depakote.

A client who has been hospitalized for two weeks for paranoia continuously complains to the staff that someone is trying to steal his clothing. What is the correct action for the nurse to take? A) Enroll the client in an exercise class to promote positive activies B) Place a lock on the client's closet to allay the client's concern C) Ignore the client's paranoid ideation to extinguish these behaviors. D) Explain to the client that his suspicions are false.

Answer: A Rationale: Diverting the client's attention from paranoid ideation and encouraging him to complete assignments can be helpful in assisting him to develop a positive self-image (A). The client's problem is not security, and (B) actually supports his paranoid ideation. (C) is not correct because ignoring the client's symptoms may lower his self-esteem. The nurse should not argue with the client about his delusions (D), and should not try to reason with the client regarding his paranoid ideation.

The nurse reviews the lab findings for a client's urine drug screen that is positive for cocaine. Which client behavior should be expected during cocaine withdrawal? A. Psychomotor impairment B. Agitation and hyperactivity C. Detachment from reality and drowsiness D. Distorted perceptions and hallucinations

Answer: A Rationale: During cocaine withdrawal, the nurse should expect (A) and a pattern of withdrawal symptoms similar to those of one who uses amphetamines. (B, C, and D) are signs and symptoms of a person high on cocaine rather than one who is experiencing withdrawal from cocaine.

A 22 year old client is admitted to the psychiatric unit from the medical unit following a suicide attempt with an overdose of diazepam (Valium). When developing the nursing care plan for this patient, which intervention would be most important for the nurse to include? A. Assist client to focus on personal strengths B. Set limits on self-defacing comments C. Remind client of daily activities in the milieu D. Assist the client to identify why he or she was self-destructive

Answer: A Rationale: Encouraging the client to focus on his or her strengths (A) helps the client become aware of positive qualities, assists in improving self-image, and aids in coping with past and present situations. Although nursing actions should assist the client in decreasing self-defacing comments (B) and informing the client of (C), these interventions are not priorities at this time. (D) is not as important as assisting the client to overcome depression, which resulted in the overdose, and asking why is nontherapuetic.

The nurse is assessing a young client admitted to the psychiatric unit for acute depression related to a recent divorce. Which statement is most indicative of a client suffering from depression? A. "I'm not very pretty or likable." B. "I've lost 20 lbs in the past month." C. "I like to keep things to myself." D. "I think everyone is out to get me."

Answer: A Rationale: Feelings of hopelessness (A) are characteristics of one who is depressed. Although (B) might be indicative of depression, further assessment may be required to rule out an organic cause before attributing the statement to depression. (C and D) are indicative of a paranoid personality.

A client who has been admitted to the psychiatric unit tells the nurse, "My problems are so bad. No one can help me." Which response would be best for the nurse to make? A. "How can I help me? Tell me more about your problems." B. "Things probably aren't as bad as they seem right now." C. "Let's talk about what is right with your life." D. "I hear your misery, but things will get better soon."

Answer: A Rationale: Offering self shows empathy and caring (A) and gives the client the opportunity to talk while the nurse listens. (B) dismisses the client's perception that things are really bad and potentially stops further communication with the client. (C) avoids the client's problems and promotes denial. "I hear your misery" (D) is an example of reflective dialogue and would be the best choice if it were not for the rest of the sentence. which offers false reassurance.

A schizophrenic client who is taking fluphenazine decanoate (Prolixin decanoate) is being discharged in the morning. A repeat dose of medication is scheduled for 20 days after discharge. The client tells the nurse about a planned vacation and will return in 18 days. Which statement by the client indicated to the nurse a need for health teaching? A. "I am going to have lots of fun at the beach and plenty of time in the sun." B. "While on vacation, I will not eat or drink anything that contains alcohol." C. "I will notify the healthcare provider if I have a sore throat or flulike symptoms" D. "I will continue to take my benztropine mesylate (Cogentin) every day"

Answer: A Rationale: Photosensitivity is a side effect of Prolixin, so the client should be instructed to avoid the sun (A). (B, C, and D) indicate accurate knowledge. Alcohol acts synergistically with Prolixin (B). A sore throat and flulike symptoms (C) are signs of agranulocytosis, which is a side effect of Prolixin. To avoid extrapyramidal symptoms, anticholinergic drugs, such as Cogentin (D), are often prescribed prophylactically with Prolixin.

A client begins taking an atypical antipsychotic medication. The nurse must provide informed consent and education about common medication side effects. Which client education will be most important? A. Maintain a balanced diet and adequate exercise B. Be sure that the diet is adequate in salt intake C. Monitor for any changes in sleep pattern D. Report any unusual facial movements

Answer: A Rationale: Several atypical antipsychotic medications can cause significant weight gain, so the client should be advised to maintain a balanced diet and adequate exercise (A). (B) is important with lithium, a mood stabilizer. (C and D) are less common than weight gain

A women brings her 48 year old husband to the outpatient psychiatric unit and tells the nurse that he has been sleepwalking, cannot remember who he is, and exhibits multiple personalities. These behaviors are often associated with which condition? A. Dissociative disorder B. Obsessive-compulsive disorder C. Panic disorder D. Posttraumatic stress syndrome

Answer: A Rationale: Sleepwalking, amnesia, and multiple personalities are examples of detaching emotional conflict from one's consciousness (A). (B) is characterized by persistent, recurrent intrusive thought or urges that are unwilled and cannot be ignored and provoke impulsive acts such as constant and repeated hand washing. (C) is an acute attack of anxiety characterized by personality disorganization. (D) is reexpeeriencing a psychologically terrifying or distressing event that is outside the usual range of human experience such as war or rape.

The nurse develops a plan of care for a client with symptoms of paranoia and psychosis. The priority nursing diagnosis is Impaired social interactions related to inability to trust. Which intervention is most important for the nurse to implement? A. Greet the client by first name during each social interaction B. Determine if the client is experiencing auditory hallucinations C. Introduce the client to peers on the unit as soon as possible D. Assign the client to a group about developing social skills

Answer: A Rationale: The most important nursing intervention is to greet the client by name (A) and provide short frequent contact to establish trust. The presence of auditory hallucinations can affect social interactions (B) but is not a priority intervention. (C and D) are effective interventions after individual rapport has been established with the client.

The nurse notes multiple burns of the arms and chest of a 2 year old Vietnamese child who is being treated for dehydration. When questioned, the child's father states that he treated the child's vomiting with the cultural practice termed coining, which resulted in burned areas. Which expected outcome statement has the highest priority? A. The child will be protected from further harm B. The family's cultural values will be respected C. The parents will express regret at harming their child D. The parents will demonstrate an ability to care for burn wounds

Answer: A Rationale: The nurse's highest priority is to ensure that no further harm befalls the child (A). (B,C,D) are also important objectives but are secondary to (A).

When planning care for the client undergoing ECT, which equipment should the nurse make available? (SATA) A. Oxygen B. Suction equipment C. Continuous passive ROM machine D. Crash Cart E. Chest tube drainage system

Answer: A, B, D Rationale: Because aspiration is a potential complication, emergency equipment such as oxygen, suction, and crash cart should be available (A, B, D). The client is only unconscious for a short period therefore there is no need for a CMP machine (C). ECT does not put the client at risk for a pneumothorax therefore a chest tube drainage system is not needed

Which topics should the nurse include in a education program for clients with schizophrenia and their families? (SATA) A. Importance of adherence to medication regimen B. Current treatment measures for substance abuse C. Signs and symptoms of an exacerbation D. Prevention of criminal activity E. Behavior modification for agression F. Chronic grief associated with long term illness

Answer: A, C, F Rationale: Medication adherence is an important component of successful rehab (A). Clients and their families also need to know the S/S of an exacerbation or relapse of the disease (C), which is frequently associated with poor medication compliance. Acknowledging the chronic sorrow associated with severe and persistent mental illness (F) helps individuals negotiate the grieving process. (B, D, and E) are not universal problems associated with schizophrenia.

A 25 year old client has suffered extensive burns and is crying during dressing change treatment. The client tells the nurse, "Please let me die. Why are you all torturing me like this? I just want to die." Which response by the nurse is best? A. "We aren't torturing you. These treatments are necessary to prevent a terrible infection." B. "I know these treatments must seem like torture to you, but we want to help you recover." C. "You have so much to live for, and all of your family members want you to live." D. "Would you like me to call the chaplain so that you can discuss your feelings privately?"

Answer: B Rationale: (B) offers empathetic response without sounding patronizing. (A) is not empathetic and is actually somewhat argumentative. The client is not asking for information as much as pleading for understanding. (C) appears as scolding and place blame on the client for wanting to die and possibly hurting the client's family members as a result. (D) might be appropriate if the nurse simply asks the client if a chaplain's visit is desired, but the nurse is dismissing the client's needs by not addressing them at the moment.

Physical examination of a 6 year old boy reveals several bite marks in various locations on his body. X-ray examination reveals healed fractures of the ribs. The mother tells the nurse that her child is always having accidents. What initial response by the nurse would be most appropriate? A. "I need to tell the healthcare provider about your child's tendency to be accident prone." B. " Tell me more about these accidents that your child has been having." C. "I need to report these injuries to the authorities because they do not seem accidental." D. "Boys this age always seem to require more supervision and can be quite accident prone."

Answer: B Rationale: (B) seeks more information using an open-ended, non threatening statement. (A) might be appropriate but is not the best answer because the nurse is being somewhat sarcastic and is also avoiding the situations by referring to the healthcare provider for resoultion. (C) is almost an attack and is jumping ahead before conclusive data can be obtained. (D) is cliche and dismisses the seriousness of the situation.

A client who is being treated with lithium carbonate for manic depression begins to develop diarrhea, vomiting, and drowsiness. Which action should the nurse take? A. Notify the healthcare provider immediately and force fluids B. Prior to giving the next dose, notify the healthcare provider of these symptoms C. Record the symptoms and continue with medication as prescribed D. Hold the medication and refuse to administer additional doses

Answer: B Rationale: Although these are expected symptoms, the healthcare provider should be notified prior to the next administration of the drug (B). Early side effects of lithium carbonate (occurring with serum lithium levels below 2 mEq/L) generally follow a progressive pattern, beginning with diarrhea, vomiting, drowsiness, and muscular weakness (C). At higher levels, ataxia, tinnitus, blurred vision, and large dilute urine output may occur. (A) will lover the lithium level. (D) is not warranted

At the first meeting of a group at a daycare center for older adults, the nurse asks one of the members what kinds of things the client would like to do with the group. The older adult shrugs and says, "You tell me. You're the leader." What would be the best response for the nurse to make? A. "Yes, I am the leader today. Would you like to be the leader tomorrow?" B. "Yes, I will be leading this group. What would you like to accomplish?" C. "Yes, I have been assigned to lead this group. I will be here for the next 6 weeks." D. "Yes, I am the leader. You seem angry about not being the leader yourself."

Answer: B Rationale: Anxiety over participation in a group and testing of the leader characteristically occur in the initial phase of the group dynamics. (B) provides information and refocuses the group defining its function. (A) is manipulative bargaining. (C) doesn't focus the group on its purpose or task. (D) is interpreting the clients feelings and is almost challenging.

A middle-aged adult was discharged from a treatment center 6 weeks ago following treatment for suicide ideation and alcohol abuse. In a follow up visit to the mental health clinic, the client complains of lethargy, apathy, irritability, and anxiety. Which question is most important for the nurse to ask? A. Are you taking prescribed antidepressants? B. How much alcohol do you consume daily? C. What seems to precipitate the anxious feelings? D. How many hours do you sleep per day?

Answer: B Rationale: First, and most importantly, the client's use of alcohol should be determined (B) because further treatment is dependent on the client's sobriety and asking how much alcohol is being consumed is a better question than asking if the client is drinking, which is a yes-no answer that does not promote dialogue. (A, C, and D) provide worthwhile information, but first the nurse should determine if the client is still drinking because all efforts to treat symptoms associated with depression are diminished if the client is still consuming alcohol.

A client on the psychiatric unit seeks out a particular nurse and imitates her mannerisms. Which defense mechanism does the nurse recognize in this client? A. Sublimination B. Identification C. Introjection D. Repression

Answer: B Rationale: Identification (B) is an attempt to be like someone or emulate the personality traits of another. (A) is substituting an unacceptable feeling for one that is more socially acceptable. (C) is incorporating the values or qualities of an admired person or group into one's own ego structure. (D) is the involuntary exclusion of painful thoughts or memories from ones awareness

Which ego defense mechanism is exhibited by a client with a phobia related to refusal to leave home? A. Denial B. Symbolization C. Fantasy D. Intellectualization

Answer: B Rationale: Symbolization (B) allows external objects to carry the internal emotional feeling through some act such as refusing to leave a safe harbour. (A) is the unconscious failure to acknowledge an event, thought, or feeling. (C) is pretending, usually of a more desirable situation. (D) is using reason to avoid emotional conflicts.

An adult client who lives in a residential facility is mentally retarded and has a history of bipolar disorder. During the past week, the client has refused to wear clothing and frequently exposed their body to other residents. Which intervention should the nurse implement? A. Establish a one to one relationship to discuss the behavior B. Redirect the client to physically demanding activities C. Encourage the client to verbalize thoughts when acting out D. Restrict social interaction with other residents in the facility

Answer: B Rationale: The client is exhibiting manic behavior related to bipolar disorder, and the nurse should redirect the client to activities that are physically demanding (B) so that energy can be expended in a socially acceptable way. Psychotic clients are not capable of (A). When exhibiting acting out behavior, the client is distracted and (C) is difficult. (D) is likely to increase manic behaviors such as mood swings and acting out behaviors

The nurse admits a client with depression to the mental health unit. The client reports difficulty concentrating, has lost 10 lbs in 2 weeks, and is sleeping 12 hours a day. Which outcome is most important for the client to meet by discharge? A. Tries to interact with a few peers and staff B. Reports feeling better and less depressed C. Sits attentively with peers in group therapy D. Easily awakens for morning medications

Answer: B Rationale: The client is experiencing symptoms of depression, and the outcome by discharge for this client would be that the client reports feeling better and less depressed (B). The client may interact with peers and staff (A) and sit attentively in groups (C) without any improvement in depression. Difficulty awakening is usually caused by the medication regimen for depression, so awakening (D) is not an indication of improvement.

Clients are preparing to leave the mental health unit for an outdoor smoke break. A client on constant observation cannot leave and becomes agitated and demands to smoke a cigarette. Which action should the nurse take first? A. Remind the client to wear the nicotine patch B. Determine if the client still needs constant observation C. Encourage the client to attend the smoking cessation group D. Explain that clients on observation cannot smoke

Answer: B Rationale: The nurse should continually reassess the need for constant observation (B) so that the client can have unit privileges such as outdoor breaks. (A and C) do not meet the client's needs and desire to smoke. (D) will cause more agitation.

A client in an acute care facility has been taking antipsychotic medications for the past 3 days with a decrease in psychotic behaviors and no adverse reactions. On the fourth day, the client experiences an increase in blood pressure and temperature and demonstrates muscular rigidity. Which action should the nurse initiate? A. Place the client on seizure precautions and monitor frequently B. Take the client's vital signs and notify the healthcare provider immediately C. Describe the symptoms to the charge nurse and document them in the client's record D. No action is required at this time because these are known side effects of her medication

Answer: B Rationale: This is an emergency situation, and the client requires immediate management in a critical care setting (B). These symptoms are descriptive of neuroleptic malignant syndrome (NMS), an extremely serious and life-threatening reaction to neuroleptic drugs. The major symptoms include fever, rigidity, autonomic instability, and encephalopathy. Respiratory failure, cardiovascular collapse, arrhythmia, and/or renal failure can result in death. (A) is not indicated in this situation. (C) doesn't consider the serious of the situation. (D) is an incorrect statement.

What instructions should the nurse include in the discharge teaching plan of a client who has recently been prescribed oxazepam (Serax)? (Select all that apply) A. Take the medication in the morning for the best results B. Do not combine this medication with alcohol C. This medication is typically used for short-term treatment D. Stop the drug immediately if sleepiness occurs E. Avoid driving or operating equipment while taking this drug

Answer: B, C, E Rationale: Harm can occur if oxazepam is taken with alcohol or other central nervous system depressants (B). Oxazepam is a benzodiazepine used for the short-term treatment of anxiety (C). Sleepiness is an expected side effect; therefore driving or operating equipment should be avoided (E). The drug should be taken in the evening because of sedation effects (A) and should be tapered, not immediately stopped, because of withdrawal effects (D).

A 45 year old male client tells the nurse that he used to believe he was Jesus Christ, but now knows he is not. Which response is best for the nurse to make? A. "Did you really believe you were Jesus Christ?" B. "I think you're getting well." C. "Others have had similar thought when under stress." D. "Why did you think you were Jesus Christ?"

Answer: C Rationale: (C) offers support by assuring the client that other have experienced similar situations. (A) is belittling. (B) is making an inappropriate judgement. You may have narrowed your choices to (C and D). However, you should eliminate (D) because it is a why question and the client doesn't know why.

An individual with a known history of alcohol abuse is admitted for emergency surgery following a motor vehicle accident. The nurse includes in the client's plan of care, "Observe for signs of delirium tremens."" Which early signs indicate that the client is beginning to have a delirium tremens? A. Abdominal cramping and watery eyes B. Depression and fatigue C. Restlessness and confusion D. Hostility and anger

Answer: C Rationale: A client experiencing alcohol withdrawal often has delirium tremens (DTs), which is characterized by progressive disorientation. Initially the client appears restless and confused (C) and develop tachycardia, tachypnea, and diaphoresis. Hallucinations, paranoia, and seizures can also occur later in the development of DTs. (A) is indicative of withdrawal from opiates such as heroin or morphine. (B) is often seen in cocaine withdrawal. (D) is most characteristic of the paranoid client.

A middle-aged client tells the nurse, "I'm again starting to feel overwhelmed and anxious with all my responsibilities. I don't know what to do." Which is the best response for the nurse to make? A. "Describe in more detail your feelings of being overwhelmed.' B. "Why don't you give up some of your commitments?" C. "What has worked for you in the past?" D. "I know, but it is important to take time for yourself."

Answer: C Rationale: A nurse can help the client solve problems by identifying past coping mechanisms that could be transferred into current situations that the client finds to be overwhelming (C). The client has already expressed some degree of hopelessness, so (A) is redundant. (B) is advice giving and may not be possible for the person and this response does not encourage the client to employ known methods of coping. (D) is also considered advice giving, wit an implied value judgement.

A client believes that his healthcare provider is an FBI agent and that his apartment is a site for slave trading. The client believe that the FBI has cameras in the apartment, so it is not safe to return there. Based on these symptoms, which class of medication is most likely to find to be prescribed for this client? A. Antianxiety B. Mood Stabilizer C. Antipsychotic D. Sedative-hypnotic

Answer: C Rationale: An antipsychotic (C) will most likely be prescribed because the client's thoughts are delusional. The client needs an antipsychotic to promote rational thought. (A) may lessen anxiety associated with the delusions, but is not the treatment of choice for altered thoughts. (B) will manage mood swings, and (D) will be prescribed for sleep.

A 35 year old client admitted to the psychiatric unit of an acute care hospital tell the nurse that someone is trying to poison her. The client's delusions are most likely related to which factor? A. Authority issues in childhood B. Anger about being hospitalized C. Low self-esteem D. Phobia of food

Answer: C Rationale: Delusional clients have difficulty with trust and have low self-esteem (C). Nursing care should be directed at building trust and promoting positive self-esteem. Activities with limited concentration and no competition should be encouraged to build self-esteem. (A, B, and D) are not specfically related to the development of delusions.

A client who recently retired is admitted to the psychiatric inpatient unit with a diagnosis of major depression. The initial nursing care plan includes the goal, "Assist client to express feelings of guilt." What is true about the goal statement referring to the client's depression? A. Implementation of the goal should be deferred until further data can be gathered B. The depression will dissipate once the client become accustomed to retirement C. Depressed clients may be unaware of guilt feelings and should be encouraged to increase self awareness D. Nursing goals should be approved by the treatment team before they are initiated

Answer: C Rationale: Depression is associated with feelings of guilt, and clients are often not aware of these feelings (C). Awareness is the first step in dealing with guilt, so the nurse's efforts should be directed toward increasing the client's awareness of feelings. Although a goal may be changed based on an evaluation of interventions to meet the goal, a goal should never be ignored (A). (B) dismisses the client's symptoms as age-related. Setting goals for the nursing care plan is a function of the nurse (D) although the nurse can collaborate with the treatment team.

A client on the psychiatric unit, diagnosed with bipolar disorder, becomes loud and shouts at one of the nurses, "You fat tub of lard, get something done around here!" What is the best initial action for the nurse to take? A. Have the staff escort the client to his room B. Tell the client that his behavior will be documented in his record C. Redirect the client by offering an activity such as playing card games D. Review the medication for an antipsychotic drug

Answer: C Rationale: Distracting the client, or redirecting him toward a constructive activity (C) prevents further escalation of the inappropriate behavior. (A) could result in escalating the abuse and might unnecessarily involve another staff member in the abusive situation. (B) may be more threatening to the client. (D) may be indicated if the behavior escalates but at this time the best initial action is (C).

A client who was admitted 2 days earlier to a drug rehab unit tells the nurse, "I'm going to do what you people tell me to so I can get out of here and get a job." What is the most accurate interpretation of this client's statement? A. The treatment program is effective and the client is highly motivated B. Defense mechanisms are being used to decrease anxiety C. Manipulation is being used to achieve the client's personal goals D. The client has insight into his behaviors so privileges should be given

Answer: C Rationale: Drug abusers tend to be manipulative so (C) is the best interpretation of the client's statement at this time in the client's treatment. He has been in treatment only 2 days which is not enough time to benefit from the program, so (A and D) are highly unlikely. Although defense mechanisms (B) are frequently used to decrease anxiety, this statement is more likely because of (C)

During a home visit, a client with schizophrenia reports hearing voices that tell the client to walk in the middle of the street. The nurse records several statements made by the client. Based on which statement should the nurse determine that the client needs hospitalization? A. "Sometimes I take an extra one of my pills when I hear the voices." B. "The voices are louder when I forget to take my medication." C. "No matter what I do, I cannot make the voices away." D. "I just try to tell the voices to stop when they bother me."

Answer: C Rationale: Hospitalization is needed if the client continues to hear voices telling the client to do things that can cause self harm (C). (A or B) do not require hospitalization unless symptoms become severe. The client should continue symptom management strategies (D) to prevent hospitalization.

A nurse working in the emergency department of a children's hospital admits a child whose injuries could have been the result of abuse. Which statement most accurately describes the nurse's responsibility in cases of suspected child abuse? A. Obtain objective data such as radiographs before reporting suspicions B. Confirm suspicions of abuse with the health care provider C. Report any case of suspected child abuse D. Document injuries to confirm suspected abuse

Answer: C Rationale: It is the nurse's legal responsibility to report all suspected cases of child abuse (C) and notifying the nurse manager or charge nurse starts the legal reporting process. (A, B, and D) delay the first step in reporting the abuse.

A child is brought to the emergency department with a broken arm. Because of other injuries, the nurse suspects that the child may be a victim of abuse. When the nurse tries to give the child an injection, the child's mother becomes very loud and shouts, "I won't leave my son. Don't you touch him! You'll hurt my child!" What is the best interpretation of the mother's statements? A. She is regressing to an earlier behavior pattern B. She is sublimating her anger C. She is projecting her feeling on the nurse D. She is suppressing her fear

Answer: C Rationale: Projection is attributing one's own thoughts, impulses, or behaviors onto another; it is the mother who is probably harming the child, and she attributing her actions to the nurse (C). The mother may be immature, but (A) is not the best description of her behavior. (B) is substituting a socially acceptable feeling for an unacceptable one. These are not socially acceptable feelings. The mother may be suppressing her fear (D) by displaying anger, but such an interpretation cannot be concluded from the data presented.

While in group therapy, a client who is diagnosed with PTSD is processing an experience from the war in Iraq when another client tips over a chair. What action should the nurse take when the client with PTSD falls to the floor in fetal position? A. Confront the client who tipped over the chair about the inconsiderate behavior B. Dismiss the other clients from the group therapy session for a 10 minute break C. Reinforce reality to the client on the floor and remove him to a quiet place D. Call a security code and medicate both clients with an antianxiety drug

Answer: C Rationale: The client who is diagnosed with PTSD is reexperiencing the traumatic experience and needs reality reassurance (confirmation that there is no danger at this time) and reduced stimuli (C). (A, B, and D) do not consider the needs of these clients at this time

A client in the critical care unit who has been oriented suddenly becomes disoriented and fearful. Assessment of vital signs and other physical parameters reveal no significant change and the nurse formulates the diagnosis, "Confusion related to ICU psychosis." Which intervention is best to implement? A) Move all machines away from the client's immediate area. B) Allay fears by teaching the client about the causes of the disease C) Cluster care so that brief periods of rest can be scheduled during the day. D) Encourage visitation by the client's family members including clients young children

Answer: C Rationale: The critical care environment confronts clients with an environment which provides stressors heightened by treatment modalities that may prove to be lifesaving. These stressors can result in isolation and confusion. The best intervention is to provide the client with rest periods (C). (A) is not practical--the machinery is often lifesaving. The client is not ready for (B). Although family and friends (D) can provide a support system to the client, visits should be limited because of the critical care that must be provided.

On admission, a depressed client tells the nurse, "I can't eat because my tongue is rubber." Which is the best action for the nurse to implement? A. Provide packaged foods for the client to eat B. Begin the client on total parenteral nutritional therapy C. Provide a well-balanced liquid diet for the client D. No action is necessary because the client will eat when hungry

Answer: C Rationale: The nurse should strive to provide a safe environment (adequate nutrition is part of a safe environment) and should not argue with the client's delusions. (C) is the least invasive while providing nutrition that doesn't argue with the client's delusion. (A) is given those with paranoid delusions. (B) is invasive and would be used as a last resort. (C) should be tried first. This client's delusion could be life threatening and should not be ignored (D).

A client is admitted with a diagnosis of depression. Which of the following characteristics is most indicative of depression? A. Grandiose ideation B. Self-destructive thoughts C. Suspiciousness of others D. Negative self image

Answer: D Rationale: A negative self image (D) is a specific indicator for depression. (A) occurs with paranoia or paranoia ideation (C). (B) may be seen in depressed clients but not always

The nurse is planning to initiate a socialization group for older residents of a long-term facility. Which information would be most useful to the nurse when planning the group? A. Each resident's length of stay at the nursing home B. A brief description of each residents family life c. The age and medication regimen of each group member D. The usual activity pattern of each group member

Answer: D Rationale: An older person's level of activity (D) is a determining factor in adjustment to aging as described in (A, B, and C) might be useful to the nurse but is not as helpful during the initiation of the socialization group. The most useful initial information would be an assessment of each individuals adjustment to the aging process.

Which behavior indicates to the nurse that a client with paranoid ideas is improving? A. Arrives on time for all activities B. Talks more openly about plans to protect his possessions C. Aggressively uses the punching bag in the gym D. Discusses his feelings of anxiety with the nurse

Answer: D Rationale: Anxious feelings increase paranoid ideation. If the client is able to discuss feelings (D), then the client is improving because of fewer paranoid ideas. (A) would indicate that a client with depression or one who is passive-aggressive is improving. (B) indicates the release of anger, and "anger turned inward" is sometimes used as a definition for depression.

On admission, a highly anxious client is described as delusional. Delusions are most likely to occur with which disorder? A. Dissociative disorders B. Personality disorders C. Anxiety disorders D. Psychotic disorders

Answer: D Rationale: Delusions are false beliefs characteristic of psychosis (D). Delusions are generally not characteristic of (A, B, and C)

A client mumbles out loud whether anyone is talking to her or not and the client also mumbles in group when others are talking. The nurse determines that the client is experiencing hallucinations. What intervention should the nurse implement? A. Respond to the client's feeling rather than the illogical thoughts B. Identify beliefs and thoughts about what the client is experiencing C. Provide the client with hope that the voices will eventually go away D. Ask the client how she has previously managed the voices

Answer: D Rationale: The nurse should promote symptom management and determine how the client previously managed the voices (D). (A and B) are interventions that are useful with clients who are experiencing delusions. (C) is important but the most important intervention is to promote symptom management

11. At the first meeting of a group of older adults at a daycare center for the elderly, the nurse asks one of the members what kinds of things she would like to do with the group. The older woman shrugs her shoulders and says, "You tell me, you're the leader." What is the best response for the nurse to make? A) Yes, I am the leader today. Would you like to be the leader tomorrow? B) Yes, I will be leading this group. What would you like to accomplish during this time? C) Yes, I have been assigned to be the leader of this group. I will be here for the next six weeks. D) Yes, I am the leader. You seem angry about not being the leader yourself.

Anxiety over participation in a group and testing of the leader characteristically occur in the initial phase of group dynamics. (B) provides information and focuses the group back to defining its function. (A) is manipulative bargaining. Although (C) provides information, it does not focus the group on its purpose or task. (D) is interpreting the client's feelings and is almost challenging. Correct Answer(s): B

60. A 22-year-old male client is admitted to the emergency center following a suicide attempt. His records reveal that this is his third suicide attempt in the past two years. He is conscious, but does not respond to verbal commands for treatment. Which assessment finding should prompt the nurse to prepare the client for gastric lavage? A) He ingested the drug 3 hours prior to admission to the emergency center. B) The family reports that he took an entire bottle of acetaminophen (Tylenol). C) He is unresponsive to instructions and is unable to cooperate with emetic therapy. D) Those with repeated suicide attempts desire punishment to relieve their guilt.

Because the client is unable to follow instructions, emetic therapy would be very difficult to implement and gastric lavage would be necessary (C). (A and B) should be considered in determining the course of treatment, but they are not the basis for determining if gastric lavage will be implemented. Medical treatments should never be used as "punitive" measures (D). Correct Answer(s): C

36. A 27-year-old female client is admitted to the psychiatric hospital with a diagnosis of bipolar disorder, manic phase. She is demanding and active. Which intervention should the nurse include in this client's plan of care? A) Schedule her to attend various group activities. B) Reinforce her ability to make her own decisions. C) Encourage her to identify feelings of anger. D) Provide a structured environment with little stimuli.

Clients in the manic phase of a bipolar disorder require decreased stimuli and a structured environment (D). Plan noncompetitive activities that can be carried out alone. (A) is contraindicated; stimuli should be reduced as much as possible. Impulsive decision-making is characteristic of clients with bipolar disorder. To prevent future complications, the nurse should monitor these clients' decisions and assist them in the decision-making process (B). (C) is more often associated with depression than with bipolar disorder. Correct Answer(s): D

Over a period of several weeks, one male participant of a socialization group at a community day care center for the elderly monopolizes most of the group's time and interrupts others when they are talking. What is the best action for the nurse to take in this situation? A) Talk to the client outside the group about his behavior during group meetings. B) Remind the client to allow others in the group a chance to talk. C) Allow the group to handle the problem. D) Ask the client to join another group.

Correct Answer(s): C Rationale: After several weeks, the group is in the working phase and the group members should be allowed to determine the direction of the group. The nurse should ignore the client's comments and allow the group to handle the situation (C). A good leader should not have separate meetings with group members (A), as such behavior is manipulative on the part of the leader. (B) is dictatorial and is not in keeping with good leadership skills. (D) is avoiding the problem. Remember, identify what phase the group is in--initial, working, or termination--this will help determine communication style.

A 25-year-old female client has been particularly restless and the nurse finds her trying to leave the psychiatric unit. She tells the nurse, "Please let me go! I must leave because the secret police are after me." Which response is best for the nurse to make? A) No one is after you, you're safe here. B) You'll feel better after you have rested. C) I know you must feel lonely and frightened. D) Come with me to your room and I will sit with you.

Correct Answer(s): D Rationale: (D) is the best response because it offers support without judgment or demands. (A) is arguing with the client's delusion. (B) is offering false reassurance. (C) is a violation of therapeutic communication in that the nurse is telling the client how she feels (frightened and lonely), rather than allowing the client to describe her own feelings. Hallucinating and/or delusional clients are not capable of discussing their feelings, particularly when they perceive a crisis.

40. A female client with obsessive-compulsive disorder (OCD) is describing her obsessions and compulsions and asks the nurse why these make her feel safer. What information should the nurse include in this client's teaching plan? (Select all that apply.) A) Compulsions relieve anxiety. B) Anxiety is the key reason for OCD. C) Obsessions cause compulsions. D) Obsessive thoughts are linked to levels of neurochemicals. E) Antidepressant medications increase serotonin levels.

Correct choices are (A, B, D, and E). To promote client understanding and compliance, the teaching plan should include explanations about the origin and treatment options of OCD symptomology. Compulsions are behaviors that help relieve anxiety (A), which is a vague feeling related to unknown fears, that motivate behavior (B) to help the client cope and feel secure. All obsessions (C) do not result in compulsive behavior. OCD is supported by the neurophysiology theory, which attributes a diminished level of neurochemicals (D), particularly serotonin, and responds to selective serotonin reuptake inhibitors (SSRI). Correct Answer(s): A, B, D, E

24. On admission, a highly anxious client is described as delusional. The nurse understands that delusions are most likely to occur with which class of disorder? A) Neurotic. B) Personality. C) Anxiety. D) Psychotic.

Delusions are false beliefs associated with psychotic behavior, and psychotic persons are not in touch with reality (D). (A, B, and C) are mental health disorders which are not associated with a break in reality, nor with hallucinations (false sensations such as hearing, or seeing) or delusions (false beliefs). Correct Answer(s): D

19. An adult male client who was admitted to the mental health unit yesterday tells the nurse that microchips were planted in his head for military surveillance of his every move. Which response is best for the nurse to provide? A) You are in the hospital, and I am the nurse caring for you. B) It must be difficult for you to control your anxious feelings. C) Go to occupational therapy and start a project. D) You are not in a war area now; this is the United States.

Delusions often generate fear and isolation, so the nurse should help the client participate in activities that avoid focusing on the false belief and encourage interaction with others (C). Delusions are often well-fixed, and though (A) reinforces reality, it is argumentative and dismisses the client's fears. It is often difficult for the client to recognize the relationship between delusions and anxiety (B), and the nurse should reassure the client that he is in a safe place. Dismissing delusional thinking (D) is unrealistic because neurochemical imbalances that cause positive symptoms of schizophrenia require antipsychotic drug therapy. Correct Answer(s): C

58. The nurse is planning the care for a 32-year-old male client with acute depression. Which nursing intervention bests helps this client deal with his depression? A) Ensure that the client's day is filled with group activities. B) Assist the client in exploring feelings of shame, anger, and guilt. C) Allow the client to initiate and determine activities of daily living. D) Encourage the client to explore the rationale for his depression.

Depression is associated with feelings of shame, anger, and guilt. Exploring such feelings is an important nursing intervention for the depressed client (B). If the client's day is filled with group activities (A) he might not have the opportunity to explore these feelings. (C) is a good intervention for the chronically depressed client who exhibits vegetative signs of depression. (D) is essentially asking the client "why" he is depressed--avoid "why's" disguised as "rationale." Correct Answer(s): B

55. A 65-year-old female client complains to the nurse that recently she has been hearing voices. What question should the nurse ask this client first? A) Do you have problems with hallucinations? B) Are you ever alone when you hear the voices? C) Has anyone in your family had hearing problems? D) Do you see things that others cannot see?

Determining if the client is alone when she hears voices (B) will assist in differentiating between hallucinations and hearing loss; this is especially important in the aging population. If the client is experiencing hallucinations, the voices will be real to her, and it is unlikely that (A) would provide accurate information. (C and D) might be good follow-up questions, but would not have the priority of (B). Correct Answer(s): B

74. A client who is diagnosed with schizophrenia is admitted to the hospital. The nurse assesses the client's mental status. Which assessment finding is most characteristic of a client with schizophrenia? A) Mood swings. B) Extreme sadness. C) Manipulative behavior. D) Flat affect.

Disinterest, and diminished or lack of facial expression is characteristic of schizophrenia and is referred to as a flat affect (D). (A) is associated with bipolar disorder. (B) is associated with depression. (C) is usually associated with personality disorders and is often seen in clients who abuse substances. Correct Answer(s): D


Set pelajaran terkait

1-4, 6, 18, 24, 31. Health Promotion, Wellness, Disease Prevention & Religion

View Set

CH 7 - Using Social Media in Business

View Set